Download as doc, pdf, or txt
Download as doc, pdf, or txt
You are on page 1of 79

1.

NUMBER SYSTEM:
Number system is a very important chapter and you will get questions from this
area in many competitive exams. We start with classification of numbers.

Types of Numbers:
1. Natural numbers (N) = 1, 2, 3, . . . .
2. Whole numbers (W) = 0, 1, 2, 3, . . . .
3. Integers (Z) = −∞ . . . −2, −1, 0, 1, 2, 3, . . .
4. Rational numbers (Q) = The numbers of the form p⁄q where q ≠ 0. Eg: 1⁄5 , 0.46,
0.333333
Other types of numbers:
1. Even numbers : Numbers which are exactly divisible by 2. These numbers are in
the format of 2n.
2. Odd numbers: Numbers which gives remainder 1 when divided by 2. These
numbers are in the format of 2n ± 1.
3. Prime numbers: The numbers which are divisible by 1 and the number itself are
primes. The least prime is 2.
4. Composite numbers: The numbers of which are divisible by more than 2
numbers.

The following rules related to Even and Odd numbers are important:
odd ± odd = even;
even ± even = even;
even ± odd = odd

odd × odd = odd;


even × even = even;
even × odd = even.

Odd (any number) = odd


even (any number) = even.
A number can be written in its prime factorization format. For example, 100 = 22 x 52.
Formula 1: The number of factors of a number N = ap x bq x cr ... = (p+1).(q+1).(r+1)...
Example: Find the number of factors of 100.
Ans: We know that 100 = 22 x 52
So number of factors of 100 = (2 +1 ).(2 +1) = 9.
In fact the factors are 1, 2, 4, 5, 10, 20, 25, 50, 100.
Sum of the factors= [(a p + 1 – 1)/ (a-1)] × [(b q+1 – 1)/ (b-1)].
Formula 2: If N=A×B×C.... then the remainder when N is divided by D is equal to the product of
the remainders when A, B, C ... are divided by D.

⇒ (N/D)R = (A/D)R × (B/D)R × (C/D)R…

Here (N/D)R means the remainder when N is divided by D.


Solved Example 1:
Find the remainder when 1201 × 1203 ×1205 × 1207 is divided by 6.
Explanation:
If you don't know the above rule, this problem is really calculation intensive.
But by applying the above rule, when 1201, 1201, 1203, 1204 divided by 6, leaves remainders 1,
3, 5, 1. The product of these remainders = 15.
When 15 is divided by 6, Remainder is 3.
Formula 3: If N=A+B+C.... then the remainder when N is divided by D is equal to the sum of
the remainders when A, B, C ... are divided by D.

⇒ (N/D)R = (A/D)R + (B/D)R + (C/D)R…


Solved Example 2:
Find the remainder when 1! + 2! + 3! + 4! + 5! + .......100! is divided by 24.
Explanation:
By applying rule 2, we divide the terms of the above expression individually, and add them to
get the final remainder. But from 4! On wards all the terms leave a remainder 0 when divided
by 24.
So the remainder = 1 + 2 + 6 + 0 + 0....... = 9.
Divisibility Rules:
2 If the last digit is even, the number is divisible by 2.
3 If the sum of the digits is divisible by 3, the number is also.
4 If the last two digits form a number divisible by 4, the number is also.
5 If the last digit is a 5 or a 0, the number is divisible by 5.
6 If the number is divisible by both 3 and 2, it is also divisible by 6.
7 Take the last digit, double it, and subtract it from the rest of the number;
if the answer is divisible by 7 (including 0), then the number is also.
8 If the last three digits form a number divisible by 8,
then so is the whole number.
9 If the sum of the digits is divisible by 9, the number is also.
1
If the number ends in 0, it is divisible by 10.
0
1 Alternately add and subtract the digits from left to right. (You can think of the first digit as
1 being 'added' to zero.)
If the result (including 0) is divisible by 11, the number is also.
Example: to see whether 365167484 is divisible by 11, start by subtracting:
[0+]3-6+5-1+6-7+4-8+4 = 0; therefore 365167484 is divisible by 11.
1 If the number is divisible by both 3 and 4, it is also divisible by 12.
2

Intermediate Divisibility Rules:


A positive integer N is divisible by
13- if 4 times the last digit of N plus the number obtained by removing the units digit of is a
multiple of 13.
17- if the last digit of N subtracted 5 times from the remaining number (excluding the units
digit) results in a number that is divisible by 17.
19- The last digit is doubled, then added to the other digits.
Finding Unit and Last two digits of an expression:
Cyclist of unit digit of an expression:
2- 2, 4, 8, 6.
3- 3, 9, 7, 1.
4- 4, 6
7- 7, 9, 3, 1.
8- 8, 4, 2, 6
9- 9, 1
For remaining digits (0, 1, 5, 6) always respective number.
Ex: (60)215= 0 unit digit
Rule-1: The last digit or unit digit even and the power is divisible by “4” then the unit place is
“6”.
Ex: (84)28= “6” unit digit.
Rule-2: The last digit or unit digit odd and the power is divisible by “4” then the unit place is “1”.
Ex: (8427)256= “1’ unit digit
Problem 1:
What is the unit digit of the expression 317171
Solution:
Here we can concentrate only on the unit digit of the base and the power. Unit digit of the base
is 7 so from the table its cyclist is 4.
Let us find the remainder when 171 is divided by 4. For the divisibility rule for the 4 is to find
the remainder of the last two digits of 171, so 71 when divided by 4 gives a remainder 3. So
from the table unit digit of 73 is 3.
Examples:
1. Find unit digit of 36 x 47 x 63 x 74 x 82 x 95.
2. Find the unit digit of (13)24 x (68)57+ (24) + (57)68 + 1234 + 5678.
3. What is the remainder when 7187 is divided by 800?
Explanation:
7187 = (74)46×73=(2401)46×343
Now 2401=2400+1
, where 2400 is perfectly divisible by 800
⇒(2401)46 is divided by 800, the remainder must be 1
So, the remainder when 7187 is divided by 800 is 1×343=343.
4. A number when divided by 296 leaves 75 as remainder. When the same number is divided by
37, the remainder will be?
Explanation:
Let the Number be Y.
Then Y = 296 q + 75
= (37 x 8) q +( 37 x 2) + 1
= 37 (8q + 2) + 1
Thus, when the number is divided by 37, the remainder is 1.
5. What least number must be subtracted from 13601, so that the number is divisible by 87?
Explanation:
87)13601(156
87
-----
490
435
-----
551
522
-----
29
-----
Required number is 29.
6. A number when divided by a divisor leaves a remainder of 24. When twice the original
number is divided by the same divisor, the remainder is 11. What is the value of the divisor?
Explanation:
Let the original number be 'a'
Let the divisor be 'd'

Let the quotient of the division of a by d be 'x'

Therefore, we can write the relation as a/d=x and the remainder is 24.
i.e., a=dx+24

When twice the original number is divided by d,2a is divided by d.


We know that a=dx+24. Therefore, 2a=2dx+48

The problem states that (2dx+48)/d leaves a remainder of 11.


2dx is perfectly divisible by d and will therefore, not leave a remainder.

The remainder of 11 was obtained by dividing 48 by d


When 48 is divided by 37, the remainder that one will obtain is 11.
Hence, the divisor is 37.
7. Find the greatest 4 digit number which is exactly divisible by 5, 4, and 9?
Explanation: L.C.M of 5, 4, and 9 is 180.
180) 9999 (55
900
-----------------------
999
900
-------------------------------------------------------
99
9999-99=9900.
8. Find the smallest 5-digit number which exactly divisible by 476
Explanation: Take smallest 5- digit number i.e. 10000
By Dividing 10000 with 476, we will get 4 as remainder.
So, 10000-4+476=10,472.
9. How can you find the number of zeros at the end of expression 15 ∗ 32 ∗ 25 ∗ 40 ∗ 75 ∗ 98 ∗
112 ∗ 125?
See, the number of zeroes at the end of any number/product depend on the number of times
10 occurs in its factorization.

Therefore, to find zeroes at the any product we can simply find 10 in its product.

Since 10=2∗5, we can further simply our task to finding number of pairs of 2 and 5 in our
product.

That being said, our product can be written as 15∗32∗25∗40∗75∗98∗112∗125


=213∗32∗59∗73

we clearly have 13 number of 2 and 9 of 5.

Given that, we can make maximum 9 pairs of 2 and 5. This means, we have 9 times 10 in our
multiplication. Which means we will have 9 zeroes at the end of the given product.
10. Find no of zeroes at the end of 700!
Formula: No. of zeroes in N! = (N/5)+ (N/25)+ (N/125)+……….+
HERE NUMERATOR IS GREATER THAN OR EQUAL TO DENOMINATOR.
No. of zeroes at the end of 700!
(700/5) + (700/25) + (700/125) + (700/625)
140 + 28 + 5+ 1 (take the quotient only) = 174.
Exercise:
1) A Certain number when divided by 95 leaves a remainder 30. What is the remainder if, the
same number be divided by 19?
(1) 8 (2) 9 (3) 10 (4) none these

2) Find the unit digit of the product of all the elements of the set which consists of all the
prime numbers greater than 2 but less than 222.
(1) 3 (2) 4 (3) 5 (4) none these
3) The remainder when (222888 + 888222) is divided by 3
(1) 0 (2) 1 (3) 2 (4) none these
4) Find the unit digit of 222 + 888222.
888

(1) 0 (2) 1 (3) 2 (4) none these


8
5) Find the remainder when 11 is divided by 7
(1) 0 (2) 1 (3) 2 (4) none these
6) The unit digit of the following expression is
(1!)1 + (2!)2 + (3!)3 + (4!)4 + (5!)5 + (6!)6 + (7!)7
(1) 5 (2) 6 (3) 7 (4) none these
7) A typist starts to type the serial numbers of candidates in a list, up to 500. Minimum how
many times he has to press the keys of numerals only?
(1) 1389 (2) 1392 (3) 1344 (4) none these
8) How many different positive integers exist between 10 & 107, the sum of whose digits is
6

equal to 2?
(1) 2 (2) 7 (3) 9 (4) 99
9) A number when divided by a divisor leaves a remainder of 24. When twice the original
number is divided by the same divisor, the remainder is 11. What is the value of the divisor?
(1) 12 (2) 13 (3) 02 ( 4) 37
10) What is the remainder when 1044 * 1047 * 1050 * 1053 is divided by 33?
(1) 27 (2) 30 (3) 21 (4) 18
11) Find the no. of zeroes at the end of the product 25x 32 x 625 x 135?s
12) Find the No. of zeroes at the end of 100!

LCM & HCF:


Introduction
1. Concept of LCM, HCF important for number theory and remainder based problems.
2. LCM is important for time and speed, time and work problems.
3. LCM is also important for circular racetracks, bells, blinking lights, etc.
4. HCF is important for largest size of tiles, largest size of tape to measure a land etc.
But before getting into LCM, HCF, let’s understand
What is Prime number?
 Consider this number: 12. This number can be found in many multiplication tables for example
 1 x 12=12.
 2 x 6 =12
 3 x 4=12
 That means, 12 has many factors (1,2,3,4,6,12). Such number is called a composite number.
 On the other hand, consider this number: 29. You cannot find it in any table except 29 x 1 =29.
Such number is called a prime number.
 Let’s make a shortlist from exam point of view
Prime Non-prime (composite)
2,3,5,7,11,13,17,19,23, 4, 6, 8, 9, 10, 12, 14,
29 15….
Now hold this prime number thought in your mind for a while.
What is LCM?
First, let’s create multiplication tables of 4 and 6.
4’s multi 6’s multi
table ple table ple
4x1 6x1
4 6
= =
4x2 6x2
8 12
= =
4x3 6x3
12 18
= =
4x4 6x4
16 24
= =
4x5 6x5
20 30
= =
4x6 6x6
24 36
= =
4x7 6x7
28 42
= =
4x8 6x8
32 48
= =
4x9 6x9
36 54
= =
 Do you see any common numbers in the multiples of 4 and 6?
 Yes I see 12, 24 and 36 are common in both tables. Let’s isolate them.
4x 1 6x 1
3= 2 2= 2
4x 2 6x 2
6= 4 4= 4
4x 3 6x 3
9= 6 6= 6
 Ok so 12, 24 and 36 are common multiples of 4 and 6. But what is the smallest of these
multiples? Answer 12 is smallest.
In the exam, we’ve no time to make such big tables to find LCM. So how to quickly find LCM of two or
three numbers? There are many tricks; the easiest one is prime-factorization. We’ll learn that in a bit, but
before that:
LCM for EXAM
1. Suppose there is a circular race track. Tarak Mehta takes 4 minutes to finish it and Jethalal takes
6 minutes to finish it. Now both of them start running from the same point at the same time in
the same direction. They’ll continue running on this track forever. So after how many minutes
will they meet for the first time on the starting point? Ans. LCM of time = LCM (4,6)=12 minutes.
They’ll meet again on the starting point after 12 minutes.
2. Two bells ring at an interval of 4 and 6 minutes respectively. After how many minutes will they
ring together? Ans LCM (4,6)
3. Two traffic lights blink at an interval of 40 and 60 seconds respectively. After how many minutes
will they link together? Ans LCM (40,60).
How to find LCM using Prime-Factorization?
Suppose in the exam, we need to find LCM of 4 and 6.
Make a table like this
Num Fact
ber ors
4
6
Now you need to find the prime factors of 4 and 6.
Num Fact
ber ors
4 2x2
6 2x3
Express it in terms of “powers”. For example 2 x 2 =2 2
Num fact
ber ors
4 22
6 2x3
Now make the third row called “LCM”.
Num fact
ber ors
4 22
6 2x3
LCM
Now write all prime numbers in this “LCM row”
Num fact
ber ors
4 22
6 2x3
LCM 2, 3
Write maximum power of each prime number
Num fact
ber ors
4 22
6 2x3
LCM 22, 3
As you can see, maximum power of 2 was 2 2 (in 4’s row).
Now multiple the numbers given in LCM row
Num
factors
ber
4 22
6 2x3
22 x 3
LCM
=12
That’s our answer. LCM (4,6)=12.
If I plot this LCM situation on a Venn Diagram, it’ll look like this:

Anyways, Let’s try a difficult one: 56 and 96.


LCM of two numbers (56, 96)
Numb Facto
ers rs
56
96
First recall, in which tables do they come? Well 56 comes in 8’s table and 96 comes in 12’s table.
Num Fact
ber ors
56 8x7
12 x
96
8
 But we need factors in “prime number” format. 12 and 8 are not prime numbers. So let’s
simplify further.
 56 = 8 x 7 = 23 x 7 (; because 8 = 4 x 2 = 2 x 2 x 2)
 96 = 12 x 8 = (4×3)x(4×2)=( 22x3) (23)=25x3 (please note you have to do this things in your head, if
you start making every calculation on a piece of paper, you’ll run out of time in the exam).
Num Fact
ber ors
23 x
56
7
96 25x3
Now let’s make the LCM row. Write all prime numbers (2,3 and7) in ascending order.
Num Fact
ber ors
23 x
56
7
96 25x3
LCM 237
Now write maximum powers of each prime number.
Num Fact
ber ors
23 x
56
7
96 25x3
25 3
LCM
7
Multiply these numbers
Num
Factors
ber
56 23 x 7
96 25x3
25
LCM
x3x7=32×21=672
So LCM (56,96)=672
let’s try finding LCM of three numbers.
LCM of three numbers: (12, 15, 20)
Approach is same. Make prime factors
Num Prime
ber factors
12 22 x 3
15 3x5
20 22 x 5
Make a new row, write all prime factors in ascending order.
Num Prime
ber factors
12 22 x 3
15 3x5
20 22 x 5
LCM 2,3,5
In the last row, Write the maximum power of those prime numbers.
Num Prime
ber factors
12 22 x 3
15 3x5
20 22 x 5
LCM 22, 3, 5
Now multiple the numbers in last row
Num Prime
ber factors
12 22 x 3
15 3x5
20 22 x 5
LCM 22x3x5=60
Therefore LCM (12,15,20)=60.
You can also look at it in following way:
 12 x 5 = 60
 15 x 4 = 60
 20 x 3 = 60.
So 60 is the least common multiple.
LCM of prime numbers
Find LCM of 7,11,13
We already know these are prime numbers. So they’ll not have any common factors. We just have to
multiply them together and we’ll get LCM. But for the sake of conceptual clarity
Numb
Factors
ers
7 7x1
11 11 x 1
13 13 x 1
1x 7 x 11 x 13
LCM
=1001
So 1001 is the answer.
LCM of co-prime numbers
 Co prime numbers are those numbers that do not have any common factors. For example, 14
and 15.
 Individually none of them is prime number because 14=2 x 7 and 15 = 3 x 5.
 But they (14 and 15) do not have any common factors. So they’re called co-prime numbers
(when they’re given together).
 Any two consecutive numbers are co-prime numbers. (e.g. 11,12 or 1548,1549).
 In case of co-prime numbers, just multiply them and you will get LCM. There is no need to find
factors. example
6 2x3
7 7
LC 2 x 3 x 7 = (6)x7
M =42
Advantages of this method?
1. Extremely fast when you’ve to find LCMs of two digit numbers for example 12,15,96.
2. And usually in time speed work, pipe-cistern type questions have number in two digits (e.g. 12,
15, 96)…so it is very easy to recall in which multiplication tables do they come.
What is HCF or GCD?
 HCF= Highest common factors.
 GCD= Greatest common divisor. Names are different otherwise they’re one and same.
 Suppose you’ve to find the HCF of (4 and 6).
 I’ll write the tables of numbers that come before 4 and 6 (i.e. 1, 2 and 3.)
1x 2x 3x
11 = 2 3
1= 1=
1x 2x 3x
22 = 4 6
2= 2=
1x 2x 3x
33 = 6 9
3= 3=
1x 2x 3x 1
44 = 8
4= 4= 2
1x 2x 1 3x 1
5= 55 = 0 5= 5
1x 2x 1 3x 1
6= 66 = 2 6= 8
1x 2x 1 3x 2
7= 77 = 4 7= 1
1x 2x 1 3x 2
8= 88 = 6 8= 4
1x 2x 1 3x 2
9= 99 = 8 9= 7
Ok, in which number’s table (1, 2 or 3) do you see both 4 and 6 reappearing?
There are two such tables 1’s table and 2’s table.
4 and 6 are common in 1’s 4 and 6 are common in 2’s
table. table.
1 x 4=4 2 x 2=4
1 x 6=6 2 x 3=6.
What does this mean?
 If I divide 4 by 1, I get zero remainder. Similarly if I divide 6 by 1, I get zero remainder. In other
words, 1 is the factor of both 4 and 6. In other words, 4 and 6 come in the table of 1.
 Similarly, If I divide 4 by 2, I get zero remainder. Similarly if I divide 6 by 2, I get zero remainder. In
other words, 2 is the factor of both 4 and 6. In other words, 4 and 6 come in the table of 2.
 Thus, 4 and 6 have two common factors (1 and 2) but highest of these common factors is 2.
Therefore HCF of (4,6)=2.
HCF for EXAM?
 What is the highest number that’ll divide 4 and 6 evenly. Answer HCF (4,6)
 There is a 4 x 6m rectangular farm. Find the length of longest tape that can measure this field.
Answer HCF (4,6)
 There is a 4x 6cm floor. Find the length of largest square tile that can be evenly laid on it. Answer
HCF (4,6)
 Two drums contain 400 and 600 liters of Indian and foreign liquor respectively. What is the
biggest measure (cup) that can measure both of them exactly? Ans. HCF (400, 600).
 A teacher has 40 pens and 60 pencils. Find maximum number of students among whom she can
distribute these items evenly.
HCF finding: Prime Factorization
In the exam, we can’t make multiplication tables of every number preceding the given numbers! So here
is the shortcut technique. We’ll use the same approach we’ve used in LCM method: prime factorization.
HCF of two numbers (4, 6)
First make prime factors of given numbers.
2
42
2
6x
3
Now, make third row: HCF and write the prime numbers that are common in both numbers.
4 22
2
6 x
3
H
C 21
F
Therefore, HCF (4,6)=2
If I’ve to plot the HCF of 4 and 6 on a Venn diagram, it’ll look like this:

HCF of three numbers (12, 24, and 36)


2
1
x
2
6
3
2
x
4
8
6
3
x
6
6
But I want them in prime format. So I’ll further simplify.
1
2 x 2 x 3=22 x 3
2
2 3 x 2 x 2 x 2=23 x
4 3
3 3 x 2 x 3 x 2=22 x
6 32
In the exam you’ve to do this in your ^head.
1 22 x
2 3
2 23 x
4 3
3 22 x
6 32
Now make a new row, write the prime numbers that are common in all of above.
1 22 x
2 3
2 23 x
4 3
3 22 x
6 32
H
C 22x3
F
in case you’re confused, let me rewrite and do it again
1
22 x 3
2
2 22 x 2
4 x3
3 22 x 3
6 x3
H
C 22x3
F
The numbers highlighted in bold are common. Therefore HCF = 2 2 x 3=12.
HCF of prime numbers (13,29)
Prime numbers do not have any common factors. So HCF of such numbers is always 1. But for the clarity
let’s do it
1
13 x 1
3
2
29 x 1
9
H
1 (because 1 is common in
C
both)
F
HCF of co-prime numbers (12,25)
Again same: 1, because co prime numbers do not have common factors.
Similarly consecutive numbers (like 456,457) do not have common factors either.
Therefore, in all such cases, HCF =1.
HCF vs LCM:
If we’ve two numbers a and b. and their HCF and LCM are given then
HCF x LCM = a x b.
But this relation only work for TWO numbers and not for more than two numbers.
Let’s understand this with an example.
You know that LCM (4,6)=12 and HCF (4,6)=2.
Right hand side (multiplication of given
Left hand side (LCM x HCF)
numbers)
12 x 2 4x6
=24 =24
So both sides match. Therefore, in case of two numbers (a and b)
LCM X HCF = a x b.
But this is not always true for three numbers. For example, Find LCM and HCF of 12,15,20. You’ll get
HCF=1 and LCM=60.
Left hand side (LCM x Right hand side (multiplication of given
HCF) numbers)
60 x 1 12 x 15 x 20
=60 =3600
In this case, both sides do not match.
HCF vs LCM: For any given numbers, their LCM is always greater than or equal to the biggest number. For
example
Numbers LCM
12,15,20 60 so greater than biggest number (20)
30. which is equal to the biggest number
15,30
(30).
Similarly, for HCF, the HCF of given numbers is always less than or equal to the smallest number. For
example
Numb
HCF
ers
12,15,
1 so it is smaller than smallest number 12
20
15. so it is equal to the smallest number
15,30
15.
Ok this is just the basic overview. In the next article, we’ll see the application of these concepts. In the
mean time, try finding LCM and HCFs of following numbers
Questi Answer (LCM,
on HCF)
91, 12 1092, 1
46, 69 138, 23
69, 97 6693, 1
63, 33 693, 3
72, 58 2088, 2
5, 84 420, 1
91, 41 3731, 1
65, 57 3705, 1
74, 12 444, 2
44, 55 220, 11
8, 28,
1400, 1
175
H.C.F. and L.C.M. of Fractions:
H.C.F= (HCF of Numerators)/( LCM of denominators)
L.C.M=( LCM of numerators) / ( HCF of denominators)
EXAMPLES:
1. The H.C.F. of two numbers is 11 and their L.C.M. is 7700. If one of the numbers is
275, then the other is:

Solution: Product of two numbers= LCM of two numbers *HCF of two numbers.

Other number = 11 x 7700 = 308


275
2. The L.C.M. of two numbers is 48. The numbers are in the ratio 2 : 3. Then sum of
the number is:
Solution: Let the numbers be 2x and 3x.

Then, their L.C.M. = 6x.

So, 6x = 48 or x = 8.

The numbers are 16 and 24.

Hence, required sum = (16 + 24) = 40.

3. Six bells commence tolling together and toll at intervals of 2, 4, 6, 8 10 and 12 seconds respectively. In
20 minutes, how many times do they toll together ?
Solution: L.C.M. of 2, 4, 6, 8, 10, 12 is 120.
So, the bells will toll together after every 120 seconds (2 minutes).
In 30 minutes, they will toll 2 + 1 = 11
together 0 times.
2
4. The greatest number of four digits which is divisible by 15, 25, 40 and 75 is:
Solution: Greatest number of 4-digits is 9999.
L.C.M. of 15, 25, 40 and 75 is 600.
On dividing 9999 by 600, the remainder is 399.
Required number (9999 - 399) = 9600.
5. The G.C.D. of 1.08, 0.36 and 0.9 is:

Solution: Given numbers are 1.08, 0.36 and 0.90. H.C.F. of 108, 36 and 90 is 18,
 H.C.F. of given numbers = 0.18.
6. The least number which should be added to 2497 so that the sum is exactly divisible by 5, 6, 4
and 3 is:
Solution: L.C.M. of 5, 6, 4 and 3 = 60.
On dividing 2497 by 60, the remainder is 37.
Number to be added = (60 - 37) = 23.
7. The smallest number which gives 7 as remainder, when divisible 12, 16, 18, 21 and 28 is:
Solution: Required number = (L.C.M. of 12,16, 18, 21, 28) + 7
= 1008 + 7
= 1015
8. Find the greatest number that will divide 43, 91 and 183 so as to leave the same remainder in
each case.
Solution: Required number = H.C.F. of (91 - 43), (183 - 91) and (183 - 43)
= H.C.F. of 48, 92 and 140 = 4.
9. Let N be the greatest number that will divide 1305, 4665 and 6905, leaving the same
remainder in each case. Then sum of the digits in N is:
Solution: N = H.C.F. of (4665 - 1305), (6905 - 4665) and (6905 - 1305)
= H.C.F. of 3360, 2240 and 5600 = 1120.
Sum of digits in N = ( 1 + 1 + 2 + 0 ) = 4
10. The greatest number which on dividing 1657 and 2037 leaves remainders 6 and 5
respectively?
Solution: Required number = H.C.F. of (1657 - 6) and (2037 - 5)
= H.C.F. of 1651 and 2032 = 127.
Exercise
1. Write the prime factorization of the greatest 3-digit number.
2. Write the prime factorization of the following numbers in exponential form:
(i) 13860 (ii) 27830 (iii) 21952.
3. Find the smallest number which must be added to 9373 so that it becomes divisible by
4.
4. Find the smallest number which must be added to 605329 so that it becomes divisible
by 9.
5. Replace the letter x in the number 8x516 by the smallest digit so that the number
becomes divisible by 6.
6. Find the H.C.F. of:
(i) 24, 60, 112 (ii) 70, 84, 336, 1260.
7. Find the least number which on adding 7 is exactly divisible by each of 15, 35 and 48.
8. Find the greatest number of four digits which is exactly divisible by each of 12, 18, 40
and 45.
9. Find the least number of five digits which is exactly divisible by each of 32, 36, 60, 90
and 144.
10. Find the H.C.F. of:
(i) 72, 126, 168 (ii) 96, 528, 2160, 3520.
11. Find the greatest number that will divide 400, 435 and 541 leaving 9, 10 and 14 as
remainders respectively.
12. Which of the following pairs of numbers are co-prime?
(i) 15, 98 (ii) 198, 429 (iii) 847, 2160
13. If the product of two numbers is 84942 and their H.C.F. is 33, find their L.C.M.
14. The product of H.C.F. and L.C.M. of two numbers is 9072. If one of the numbers is 72,
find the other number.
15. The H.C.F. and L.C.M. of two numbers are 12 and 5040 respectively If one of the
numbers is 144, find the other number.
Percentages:
Important Concepts and Formulas - Percentage
Percent means for every 100. So, when percent is calculated for any value, it means that we
calculate the value for every 100 of the reference value. percent is denoted by the symbol %.
For example, x
Percent is denoted by x%
1. X%=x/100

Example:
25%=25/100=14

2. X/y=(x/y×100) %
Example:
1/4= (1/4×100) %=25%

3. If the price of a commodity increases by R%, the reduction in consumption so as not to


increase the expenditure
= [[R/ (100+R)] ×100] %

4. If the price of a commodity decreases by R%, the increase in consumption so as not to


decrease the expenditure
= [(R/ (100−R)) ×100] %

Formula 1:
A is what percentage of B? ⇒ (A/B) ×100
Formula 2:
A is how much percent greater than B? ⇒ [(A−B)/B] ×100

Formula 3:
A is how much percent less than B? ⇒ [(B−A)/B] ×100

Formula 4:
A %( B) =B %( A)

Formula 5:
If a number K got increased by A% and B% successively then the final percentage is given by
(A+B+(AB/100))%
Note1: If decreased then substitute +A% with –A

Calculating a percent change:

Examples:
1. The wages earned by Robin is 30% more than that earned by Erica. The wages earned by
Charles is 60% more than that earned by Erica. How much % is the wages earned by Charles
more than that earned by Robin?
Explanatory Answer:

Let the wages earned by Erica be 100

then, wages earned by Robin and Charles will be 130 and 160 respectively.

Charles earns 30 more than Robin who earns 130.

Therefore, Charles' wage is = 23.07%


2. In an election contested by two parties, Party D secured 12% of the total votes more than
Party R. If party R got 132,000 votes, by how many votes did it lose the election?
Explanatory Answer:

Let the percentage of the total votes secured by Party D be x%


Then the percentage of total votes secured by Party R = (x - 12)%

As there are only two parties contesting in the election, the sum total of the votes secured by
the two parties should total up to 100%

i.e., x + x - 12 = 100
2x - 12 = 100
or 2x = 112 or x = 56%.

If Party D got 56% of the votes, then Party got (56 - 12) = 44% of the total votes.

44% of the total votes = 132,000

i.e., = 132,000

=> T= = 300,000 votes.

The margin by which Party R lost the election = 12% of the total votes

= 12% of 300,000 = 36,000.

3. The difference between the value of a number increased by 12.5% and the value of the
original number decreased by 25% is 30. What is the original number?

Explanatory Answer:

Let the original number be x.

Let A be the value obtained when x is increased by 12.5%.

Therefore, A = x + 12.5% of x

Let B be the value obtained when x is decreased by 25%.

Therefore, B = x - 25% of x

The question states that A - B = 30

i.e., x + 12.5% of x - (x - 25% of x) = 30

x + 12.5% of x - x + 25% of x = 30

37.5% of x = 30

x= = 80

4. What is the % change in the area of a rectangle when its length increases by 10% and its
width decreases by 10%?
Explanatory Answer:

Whenever you encounter problems like this, use a numerical example and then proceed.

For ease of computation, it is safe in most cases, to assume the length to be 100 units and the
width to be 100 units.

(Remember, a square is a rectangle too and the problem works the same way when you assume
different values for length and width. Computation becomes a bit tedious with different values
for length and width)

Area of a rectangle = length * width = 100 * 100 = 10,000 sq units.

When the length increases by 10%, the new length becomes 110 units.

And as the width decreases by 10%, new width becomes 90 units.

Therefore, New area = 110 * 90 = 9900 sq units.

New area is 100 sq units lesser than the original area.

% change in area = ((change in area)/(original area)) * 100

= (100/10,000)*100 = 1% decrease in area.

5. The price of petrol increased by 25% and so a person reduced his consumption by 25%. What
percentage is the rise or fall in the expenditure incurred by him on petrol?

Explanatory Answer:

Method 1: Use formula (A+B+ (AB/100)) % (Note1: If decreased then substitute +A% with –A)

(25−25− (25×25)/100)%⇒−625/100%⇒−6.25%

So, reduction of 6.25% in his expenditure.

6. 15 liters of a mixture contains 20% alcohol and rest water. If 3 liters of water is added in it, per
cent of alcohol in the new mixture will be?

Explanatory Answer:

Alcohol in the mixture = 1/5 x 15 = 3 liters


Mixture becomes (15 + 3) liters = 18 liters on adding 3 liters of water.
Therefore, Percentage of alcohol in the new mixture = 3/18=1/6 = 16.67%
7. A person saves 10% of his income. If his income is increased by 20% and he saves 15% of the
new income, by what percent his savings will increase?
Explanatory Answer:

Let previous income = Rs. 100


Previous savings = 10% of Rs. 100 = Rs. 10
Increased income = Rs. 100 + 20% of Rs. 100 = Rs. 120
Increased savings = 15% of Rs. 120 = Rs. 18
Therefore, Increase in savings = Rs. 18 - Rs. 10 = Rs. 8
Therefore, Percent increase in savings = 8/10 x 100 = 80%

8. Sum of 5% of a number and 9% of other number is equal to sum of 8% of first number and
7% of the second number. Find ratio between the numbers.
Explanatory Answer:

Let the numbers are x and y.


Then 5% of x + 9% of y = 8% of x + 7% of y
Therefore, 3% of x = 2% of y
Therefore, x : y = 2 : 3

9. In an examination, 50% students failed in English and 40% in Math and 15% students failed in
both subjects. If 200 students passed in both the subjects, find the number of students
appeared in the exam.
Explanatory Answer:

Students failed in English only = (50 - 15)% = 35%


Students failed in Math only = (40 - 15)% = 25%
Students failed in both subjects = 15%
Therefore, Students failed in either or both subjects = 35 + 25 + 15 = 75%
Therefore, Students passed in both subjects = (100 - 75)% = 25%
But students passed = 200 (i.e. 8 times of 25)
Therefore, Students appeared = 8 x 100 = 800.

10. Sameer spends 40% of his salary on food articles and (1/3) rd of the remaining on transport.
If he saves Rs.450 per month which is half of the balance after spending on food items and
transport, what is his monthly salary?
Explanatory Answer:

Suppose, salary = Rs.100


Expenditure on food = Rs.40
Balance = Rs.60
Expenditure on transport
= 1/3×60=Rs.20
Now balance = Rs.40
Saving = Rs.20
If saving is 20, salary = Rs.100
If saving is 450,
salary = Rs.((100/20)×450)=Rs.2250.

Exercise:

1. Two students appeared at an examination. One of them secured 9 marks more than the other
and his marks was 56% of the sum of their marks. What are the marks obtained by them?

2. Evaluate 28% of 450 + 45% of 280?


3. An inspector rejects 0.08% of the meters as defective, How many meters he examine to reject
2 meters?
4. In an election between two candidates, 75 % of the voters cast their votes, out of which 2% of
the votes were declared invalid. A candidate got 9261 votes which were 75% of the total valid
votes. Find the total number of votes?
5. In a town, the population was 8000. In one year, the male population increased by 10% and
the female population increased by 8% but the total population increased by 9%. The number of
males in the town was?
6. Rahul's Mathematics test had 75 problems, 10 arithmetic, 30 algebra, 35 geometry problems.
Although he answered 70% of arithmetic, 40% of arithmetic and 60% of geometry problems
correctly, still he got less than 60% problems right. How many more questions he would have to
answer more to get passed?
7. Total number of boys and girls in a school is 150. If the number of boys is x, then girls become
x% of the total number of students. The number of boys is?
8. Due to an increase in 30% in the price of eggs, 3 eggs less are available for Rs. 7.80. Find the
present rate of eggs per dozen?
9. A sample of 50 liters of glycerin is found to be adulterated to the extent of 20%. How much
pure glycerin should be added to it so as to bring down the percentage of impurity to 5%?
10. The cost of an apple is twice that of a banana and the cost of a banana is 25% less than that
of guava. if the cost of each type of fruit increases by 10%, then the percentage increase in the
cost of 4 bananas, 2 apples, and 3 guavas is?
Important Formulas - Profit and Loss
Cost price (CP) is the price at which an article is purchased.
Selling price (SP) is the price at which an article is sold.
If SP > CP, it is a profit or gain
If CP > SP, it is a loss.
Gain or Profit = SP – CP
Loss = CP – SP
Loss or gain is always reckoned on CP
Profit Percentage (Profit %) = (Profit/CP)
×100= [(SP - CP)/CP] ×100.
Loss Percentage (Loss %) = (Loss/CP) ×100= [(CP - SP)/CP] ×100.
In the case of a gain or profit,
SP= [(100+Gain %) /100] ×CP

CP= [100/ (100+ Gain %)] ×SP


In the case of a loss,
SP=[(100−Loss%)/100]×CP

CP=[100/(100−Loss%)]×SP
If an article is sold at a gain of 20%, then SP = 120% of CP
If an article is sold at a loss of 20% then SP = 80% of CP

Examples:

Person incurs a loss of 5% be selling a watch for Rs. 1140. At what price should the watch be
sold to earn 5% profit?
Solution:
Let the new S.P. be x, then.
(100 - loss%):(1st S.P.) = (100 + gain%):(2nd S.P.)
=>(95/1140=105/x)=>x=1260
A book was sold for Rs 27.50 with a profit of 10%. If it were sold for Rs. 25.75, then would
have been percentage of profit and loss?
Solution: Please remember
S.P =((100+gain%)/100∗C.P) So, C.P. = ((100/110)∗25.75)
When S.P. = 25.75 then Profit=25.75−25=0.75
Profit %=( 0.75/25) ∗100=3%.
Goods are purchased for Rs. 450 and one-third is sold at a loss of 10%. At what profit per
cent should the remainder be sold so as to gain 20% on the whole transaction?
Solution: Applying weighted average, one-third of quantity there is a loss of 10% (or a profit
of –10%) and balances two-thirds gives a profit of x%.
Hence, overall profit is given by, 1/3(−10%) + 2/3(x %) = 20 % thus x = 35 %.
A man sells an article at a profit of 25%. If he had bought it at 20% less and sold it for Rs.
10.50 less, he would have gained 30%. Find the CP of the article.

Solution: Let the cost price = x ; Selling Price = 125%(x)


New Cost Price = 80%(x) ; New SP = 125%(x) – 10.50
But new SP = 130% of new CP =130%(80%x)
Therefore, 130%(80%x) = 125%(x) – 10.50 ⇒104%(x)=125%(x)−10.50
⇒21%(x)=10.50 ⇒x=10.50×(100/21)⇒Rs.50
Alternate Method:
Let the CP be Rs. 100. So, SP is Rs. 125.
The new CP is Rs. 80. So the new SP =130 %( 80) = Rs. 104
so the difference of SP’s = Rs. 21.
Now, if the difference is 21, CP is 100
So, when the difference is 10.5, the CP is (10.5/21)×100⇒Rs.50.
A man gains 20% by selling an article for a certain price. If he sells it at double the price, the
percentage of profit will be?
Explanation:
Let the C.P. = x,
Then S.P. = (120/100)x = 6x/5
New S.P. = 2(6x/5) = 12x/5

Profit = 12x/5 - x = 7x/5

Profit% = (Profit/C.P.) * 100


=> (7x/5) * (1/x) * 100 = 140 %
The cost price of 20 articles is the same as the selling price of x articles. If the profit is 25%
then determine the value of x.
Explanation:
Let the cost price 1 article = Re 1
Cost price of x articles = x
S.P of x articles = 20

Gain = 20 -x
25=((20−x)/x∗100)=>2000−100x=25x=>x=16.

In a certain store, the profit is 320% of the cost. If the cost increases by 25% but the selling
price remains constant, approximately what percentage of the selling price is the profit?
Explanation:
Let C.P = Rs. 100.
Then, Profit = Rs. 320,
S.P. = Rs. 420.

New C.P. = 125% of Rs. 100 = Rs. 125


New S.P. = Rs. 420.

Profit = Rs. (420 - 125) = Rs. 295

Required percentage = (295/420) * 100 = 70 % (approx).


A shopkeeper cheats to the extent of 10% while buying and selling, by using false weights.
His total gain is.
Explanation:
Gain % = %
= %
= %
= 21%
A retailer buys product from a shopkeeper at discount of 40% on the list price (marked
price) and sells them to the customer at a discount of 25% on the list price. What is his profit
percentage?
Explanation:
Let the list price be Rs.100.
Therefore, the retailer is buying the products at Rs.60 and selling it to the customer at Rs.75,
earning a profit of Rs.15.
Therefore, his percentage is = 25%.
10.A women goes to market with Rs.500 to buy oranges. The prices of the oranges have
decreased by 10% so he could buy 2kg more with the amount she had. What was the
original price of the oranges?
Explanation:
If the price of the oranges decreases by 10%, the woman would save 10% of the money that
is Rs.50 to buy the same amount as before. Now in Rs50 she can buy 2 kg more, therefore
the current price of 1 kg of oranges is Rs25.Now this current price is after 10% reduction.
Therefore, the original price =
Exercise:
A woman buys apples at 15 for a rupee and the same number at 20 a rupee. She mixes and
sells them at 35 for 2 rupees. What is her gain per cent or loss per cent?
A reduction of 10% in the price of sugar enables a man to buy 25 kg more for Rs. 225. What
is the original price of sugar (per kilogram)?
A dishonest merchant professes to sell his goods at cost price, but uses a weight of 900
grams for one kg. Weight. What is his profit per cent?
A shopkeeper buys some pens. If he sells them at Rs. 13 per pen, his total loss in Rs. 150 but
on selling them at Rs. 15 per pen, his total gain is Rs. 100. How many pens did he sell?
Sam buys 10 apples for $1. At what price should he sell a dozen apples if he wishes to make
a profit of 25%?
By selling an article at 80% of its marked price, a merchant makes a loss of 12%. What %
profit will the merchant make if the article is sold at 95% of its marked price?
While selling a watch, a shop-keeper gives a discount of 5%. If he gives a discount of 7%, he
earns Rs.15 less as profit. The marked price of the watch is?
A shop-keeper earns a profit of 12% on selling a book at 10% discount on the printed price.
The ratio of the cost price and the printed price of the book is?
Subhash purchased a tape recorder at 910th of its selling price and sold it at 8% more than
its S.P. His gain is?
A person bought an article and sold it at a loss of 10%. If he had bought it for 20% less and
sold it for Rs.55 more, he would have had a profit of 40%. The C.P of the article is?
Simple interest & Compound interest:
Important Formulas - Simple Interest
Introduction

Money is not free and it costs to borrow the money. Normally, the borrower has to pay an
extra amount in addition to the amount he had borrowed. i.e, To repay the loan, the
borrower has to pay the sum borrowed and the interest.
Lender and Borrower

The person giving the money is called the lender and the person taking the money is the
borrower.

Principal (sum)

Principal (or the sum) is the money borrowed or lent out for a certain period. It is denoted
by P.
Interest

Interest is the extra money paid by the borrower to the owner (lender) as a form of
compensation for the use of the money borrowed.
Simple Interest (SI)

If the interest on a sum borrowed for certain period is calculated uniformly, it is called
simple interest(SI).
Amount (A)

The total of the sum borrowed and the interest is called the amount and is denoted by A
The statement "rate of interest 10% per annum" means that the interest for one year on a
sum of Rs.100 is Rs.10. If not stated explicitly, rate of interest is assumed to be for one year.
Let Principal = P, Rate = R% per annum and Time = T years. Then

Simple Interest, SI = (PTR/100).


Important Formulas - Compound Interest
 Introduction

We have already seen simple interest in detail. Another way of calculating interest is the
compound interest method.
In compound interest method, interest for each period is added to the principal before
interest is calculated for the next period. In other words, the principal grows as the
interest gets added to it.
Some Basic Formulas
Let Principal = P, Rate = R% per annum, Time = T years and A = Amount due after T years
i. Case 1: When interest is compounded annually
A = P (1+(R/100)) T

ii. Case 2: When interest is compounded half-yearly

A = P (1+ ((R/2)/100)) 2T
iii. Case 3: When interest is compounded quarterly

A = P (1+ ((R/2)/100)) 4T
iv. Case 4: When interest is compounded annually, but time is in fraction, say 4(1/3)
years

A = P (1+(R/100)) 4× (1+ (1/3) R/100)


v. Case 5: When rates are different for different years, say R1%, R2% and R3% for 1st,
2nd and 3rd year respectively.

A = P (1+(R1/100))(1+(R2/100))(1+(R3/100))
 Wherever the term compound interest is used without specifying the period in which
the interest is compounded, it is assumed that interest is compounded annually.
 Compound Interest (CI) = A - P
 Simple Interest and Compound Interest for 1 year at a given rate of interest per annum
will be equal.
Examples:
1. The interest earned by Rs.4800 in 2 years and 3 months at the rate of 8.5%p.a. simple interest
is?
Use formula SI = (PTR/100).
P= 4800, T= 2 years 3months= 2(1/4) years, R= 8.5% p.a
2. The simple interest on a sum of money will be Rs. 600 after 10 years. If the principle is trebled
after 5 years, what will be the total interest at the end of the tenth year?

Explanation: We know that interest is directly proportional to time and principal. If the total
interest for 10 years is Rs.600, It is Rs.300 for the first 5 years. Now the principal trebled after 5
years. So we get 3 times more interest for the next 5 years. So instead of Rs.300 we get Rs.900.
So total interest = Rs.300 + Rs.900 = Rs.1200.
3. Rs. 600 amounts to Rs. 735 in 5 years at a certain rate of Simple interest. If the rate of interest
is increased by 2%, what will be the amount then?
Explanation: There is no need of calculating original rate of interest in this case. We can just
calculate the difference generated by the increment of 2% interest rate.
Increase in simple interest = 2% on Rs. 600 for 5 years
= (600×2×5)/100 = 6 x 2 x 5 = Rs. 60
Therefore Amount = Original amount + Extra interest
= Rs. 735 + Rs. 60 = Rs. 795.
4. What is the present worth of Rs. 132 due in 2 years at 5% simple interest per annum?
Explanation:
Let the present worth be Rs. x
Then, S.I.= Rs.(132 - x)

=› (x*5*2/100) = 132 - x

=› 10x = 13200 - 100x


=› 110x = 13200

x= 120
5. A father left a will of Rs.35 lakhs between his two daughters aged 8.5 and 16 such that they
may get equal amounts when each of them reach the age of 21 years. The original amount of
Rs.35 lakhs has been instructed to be invested at 10% p.a. simple interest. How much did the
elder daughter get at the time of the will?
Explanation:
Let Rs. x be the amount that the elder daughter got at the time of the will. Therefore, the
younger daughter got (3,500,000 - x).
The elder daughter’s money earns interest for (21 - 16) = 5 years @ 10% p.a simple interest.
The younger daughter’s money earns interest for (21 - 8.5) = 12.5 years @ 10% p.a simple
interest.
As the sum of money that each of the daughters get when they are 21 is the same,

6. The compound interest on rs.30000 at 7% per annum is Rs.4347. The period is?
Explanation:
Amount = Rs.(30000+4347) = Rs.34347
Let the time be n years
Then,

 n = 2years
7. The difference between compound interest and simple interest on a sum for two years at 8%
per annum, where the interest is compounded annually is Rs.16. if the interest were
compounded half yearly; the difference in two interests would be nearly?
Explanation:
For 1st year S.I =C.I.
Thus, Rs.16 is the S.I. on S.I. for 1 year, which at 8% is thus Rs.200
i.e. S.I on the principal for 1 year is Rs.200

Principle = = Rs.2500
Amount for 2 years, compounded half-yearly

C.I = Rs.424.64

Also,
Hence, [(C.I) - (S.I)] = Rs. (424.64 - 400) = Rs.24.64.
8. The compound interest on a sum of money for 2 years is rs.832 and the simple interest on the
same sum for the same period is rs.800 .the difference between the compound interest and
simple interest for 3 years
Explanation:
Difference in C.I and S.I in 2years =Rs.32
S.I for 1year =Rs.400
S.I for Rs.400 for one year =Rs.32
Rate= [100*32)/ (400*1) %=8%
Difference between in C.I and S.I for 3rd year
=S.I on Rs.832= Rs. (832*8*1)/100=Rs.66.56.
9. The present worth of Rs.169 due in 2 years at 4% per annum compound interest is?
Explanation:
Present Worth

=
= Rs.156.25
10. On a sum of money, the simple interest for 2 years is Rs. 660, while the compound interest is
Rs.696.30, the rate of interest being the same in both the cases. The rate of interest is?
Explanation:
Difference in C.I and S.I for 2 years
Rs (696.30-660)=Rs. 36.30.
S.I for one year’s = Rs330.
S.I on Rs.330 for 1 year =Rs. 36.30
Rate= (100x36.30/330x1) %= 11%
Exercise:
1. A man lent Rs. 2000 - partly at 5% and the balance at 4%. If he receives Rs. 92 towards annual
interest, find the amount lent at 5%.
2. The rate of interest on a sum of money is 4% per annum for the first 2 years, 6% per annum
for the next 4 years and 8% per annum for the period beyond 6 years. If the simple interest
occurred by the sum for a total period of 9 years is Rs.1120, what is the sum?
3. Two equal amounts of money are deposited in two banks, each at 15% per annum, for 3.5
and 5 years. If the difference between their interests is Rs.144, each sum is?
4. A sum was put at simple interest at a certain rate for 10 years. Had it been put at 5% higher
rate, it would have fetched Rs.600 more. What was the Sum?
5. A certain sum of money at simple interest amounts to Rs.1260 in 2 years and to Rs.1350 in 5
years. The rate percent per annum is?
6. Find the amount for Rs. 6000 at 10% per annum, compounded semi-annually for 2 years?
7. At what rate per cent of compound interest, a sum of Rs. 2000 will amount to Rs. 2662 in 3
years?
8. A man borrows Rs. 2100 and undertakes to pay back with compound interest @ 10% p.a. in 2
equal yearly installments at the end of first and second year. What is the amount of each
installment?
9. A father wants to divide Rs. 5100 between his two sons, Mohan and Sohan who are 23 and
24 at present. Divide the amount in such a way that if their shares are invested at compound
interest @ 4% p.a. they will receive equal amount on attaining the age of 26 years. Find
Mohan's share?
10. A sum of money becomes Rs.6690 after three years and Rs.10,035 after 6 years on
compound interest. The sum is?

Ratios & Proportions:


1. Ratio:

The ratio of two quantities a and b in the same units, is the fraction and we write it as a : b.
In the ratio a : b, we call a as the first term or antecedent and b, the second term or consequent.
5
Eg. The ratio 5 : 9 with antecedent = 5, consequent =
/
represents 9.
9

Rule: The multiplication or division of each term of a ratio by the same non-zero number does
not affect the ratio.
Eg. 4 : 5 = 8 : 10 = 12 : 15. Also, 4 : 6 = 2 : 3.
2. Proportion:
The equality of two ratios is called proportion.
If a : b = c : d, we write a : b :: c : d and we say that a, b, c, d are in proportion.
Here a and d are called extremes, while b and c are called mean terms.
Product of means = Product of extremes.
Thus, a : b :: c : d (b x c) = (a x d).
3. Fourth Proportional:
If a : b = c : d, then d is called the fourth proportional to a, b, c.
Third Proportional:
a : b = c : d, then c is called the third proportion to a and b.
Mean Proportional:
Mean proportional between a and b is ab.
4. Comparison of Ratios:
We say that (a : b) > (c : d)
a c

b d
5. Compounded Ratio:
The compounded ratio of the ratios: (a : b), (c : d), (e : f) is (ace : bdf).
6. Duplicate Ratios:
Duplicate ratio of (a : b) is (a2 : b2).
Sub-duplicate ratio of (a : b) is (√a : √b).
Triplicate ratio of (a : b) is (a3 : b3).
Sub-triplicate ratio of (a : b) is (a1/3 : b1/3).
EXAMPLES:

1. Two numbers are respectively 20% and 50% more than a third number. The ratio of the two
numbers?
Solution: Let the third number be 100
So, first number is 120 and second number is 150
Ratio is 120:150= 4:5

2. A sum of money is to be distributed among A, B, C, D in the proportion of 5 : 2 : 4 : 3. If C gets Rs. 1000


more than D, what is B's share?
Solution: Let the shares of A, B, C and D be Rs. 5x, Rs. 2x, Rs. 4x and Rs. 3x respectively.
Then, 4x - 3x = 1000
x = 1000.
B's share = Rs. 2x = Rs. (2 x 1000) = Rs. 2000.
3. Seats for Mathematics, Physics and Biology in a school are in the ratio 5 : 7 : 8. There is a proposal to
increase these seats by 40%, 50% and 75% respectively. What will be the ratio of increased seats?
Solution: Originally, let the number of seats for Mathematics, Physics and Biology be 5x, 7x and 8x
respectively.
Number of increased seats are (140% of 5x), (150% of 7x) and (175% of 8x).
Required ratio is=> 2: 3 : 4.
4. A sum of Rs. 350 made up of 110 coins, which are of either Re. 1 or Rs. 5 denomination. How many
coins are of Rs. 5?
Solution: Let, all the coins are of Re. 1 denomination.
Then, total value of 110 coins = 110 x 1 = Rs. 110 which is short from Rs. 350 by Rs. 350 - Rs. 110 = Rs.
240.
Now, replacing 1 one-rupee coin with five-rupee coin mean Rs. 4 extra.
Therefore, Five rupee coins = 240/4 = 60 coins
5. One year ago the ratio between varun's and arun's salary was 3:4. The ratio's of their individual
salaries between last year's and this year's salaries are 4:5 and 2:3 respectively. At present the total of
their salary is Rs.4290. The salary of Varun now is :
Explanation:
Let the salaries of Varun and Arun one yer before be 12x and 16x.
Given that varun's last year and present year salary are in the ratio 4 : 5 so his present salary = 5/4(12x) =
15x
Also Arun's last year and present year salary are in the ratio 2 : 3 so his present salary = 3/2(16x) = 24x
But given that sum of the salaries 15x + 24x = 39x = 4290 ⇒ x = 110
Varun's present salary = 15x = 15 x 110 = 1650
6. A right cylinder and a right circular cone have the same radius and the same volume. The ratio of the
height of the cylinder to that of the cone is :
Explanation:
Let the heights of the cylinder and cone be h and H respectively. Then,
πr2h=1/3πr2H
h/H=1/3
So, their heights are in the ratio 1 : 3
Exercise:
1.In a class, the number of boys is more than the number of girls by 12% of the total strength. The ratio
of boys to girls is :
2. If a:b = 2:3 and b:c = 4:3, then find a:b:c?
3. If A:B = 2:3, B:C = 4:5 and C:D = 6:7, then find the value of A:B:C:D?
4. if x:y = 1:3, then find the value of (7x+3y):(2x+y)?
5. Rs. 120 are divided among A, B, C such that A's share is Rs. 20 more than B's and Rs. 20 less than C's.
What is B's share?
6. Three friends Akhil, Balu and Chiru divide 1105 amongst them in such a way that if 10, 20 and 15 are
removed from the sums that Akhil, Balu and Chiru received respectively, then the share of the sums that
they got will be in the ratio of 11 : 18 : 24. How much did Chiru receive?
7. In what ratio should a 20% methyl alcohol solution be mixed with a 50% methyl alcohol solution so
that the resultant solution has 40% methyl alcohol in it?
8. A and B together have Rs. 1210. If 4/15 of A's amount is equal to 2/5 of B's amount. How much
amount B have.
9. If 40% of a number is equal to two-third of another number, what is the ratio of first number to the
second number.
10. Rs.5625 is divided among A, B and C so that A may receive 12 as much as B and C together receive, B
receives 14 of what A and C together receive.
The share of A is more than that of B by ?
Problems on ages:
Important points on "Problems on Ages" :
1. If the current age is x, then n times the age is nx.
2. If the current age is x, then age n years later/hence = x + n.
3. If the current age is x, then age n years ago = x - n.
4. The ages in a ratio a : b will be ax and bx.
Examples:
1. One year ago Jaya was four times as old as her daughter Nikitha. Six years hence, Mrs.Jaya's age will
exceed her daughter's age by 9 years. The ratio of the present ages of Jaya and her daughter is :
Explanation:
Let Nikitha's age 1 year ago = x
Then Jaya's age 1 year ago = 4x
After 6 years their ages are 4x+7, x+7
(4x+7) - (x+7) = 9 or x = 3
Present age of Jaya = (12+1) years = 13 years
Present age of Nikitha = (3+1) years = 4 years
Ratio of their ages = 13 : 4
2. The sum of ages of 5 children born at the intervals of 3 years each is 50 years. What is the age of the
youngest child?
Explanation: Let the ages of children be x, (x + 3), (x + 6), (x + 9) and (x + 12) years.
Then, x + (x + 3) + (x + 6) + (x + 9) + (x + 12) = 50
5x = 20
x = 4.
 Age of the youngest child = x = 4 years.
3. Raju age after 15 years will be 5 times his age 5 years back, What is the present age of Raju?
Explanation: Clearly,
x+15 = 5(x-5)
<=> 4x = 40 => x = 10
4. Present ages of Sameer and Anand are in the ratio of 5 : 4 respectively. Three years hence, the ratio of
their ages will become 11 : 9 respectively. What is Anand's present age in years?
Explanation: Let the present ages of Sameer and Anand be 5x years and 4x years respectively.
(5x + 3)/(4x + 3)= 11/9
9(5x + 3) = 11(4x + 3)

45x + 27 = 44x + 33

45x - 44x = 33 - 27

x = 6.

Anand's present age = 4x = 24 years.

5. A father is twice as old as his son. 20 years ago the age of the father was 12 times the age of the son.
The present age of the father is ?
Explanation:
Let son's age = x. Then, father's age = 2x.
12(x-20)=(2x-20)⇒x=22
Father's present age = 44 years
6. Ages of two persons differ by 16 years. If 6 year ago, the elder one be 3 times as old the younger one,
find their present age?
Explanation:
Let the age of younger person is x,
Then elder person age is (x+16)
=> 3(x-6) = (x+16-6) [6 years before]
=> 3x-18 = x+10
=> x = 14.
So other person age is x + 16 = 30
7. The present ages of three persons in proportions 4 : 7 : 9. Eight years ago, the sum of their ages was
56. Find their present ages (in years)?
Explanation:
Let their present ages be 4x, 7x and 9x years respectively.
Then, (4x - 8) + (7x - 8) + (9x - 8) = 56
20x = 80
x = 4.
 Their present ages are 4x = 16 years, 7x = 28 years and 9x = 36 years respectively.
8. Three years ago the average age of A and B was 18 years. With C joining them, the average becomes
22 years. How old is C now ?
Explanation:
Sum of ages of A and B, 3 years ago = (18×2)=36 years. Sum of ages of A,B and C, now = (22×3)=66 years.
Sum of ages of A and B, now =(36+6) years=42 years
C's age = (66-42)years = 24 years
9. The sum of the ages of a father and son is 45 years. Five years ago, the product of their ages was four
times the father’s age at that time. The present age of father and son?
Explanation:
Let sons age = x years. Then fathers age = (45 - x)years.
(x—5)(45—x—5) = 4(45- x - 5) hence (x—5) = 4 so x = 9
Their ages are 36 years and 9 years.
10. The age of father 10 years ago was thrice the age of his son. Ten years hence, father's age will be
twice that of his son. The ratio of their present ages is:
Explanation:
Let the ages of father and son 10 years ago be 3x and x years respectively.
Then, (3x + 10) + 10 = 2[(x + 10) + 10]
3x + 20 = 2x + 40
x = 20.
 Required ratio = (3x + 10) : (x + 10) = 70 : 30 = 7 : 3.
Exercise:
1. The age of Rekha is twelve times that of her daughter Avani. If the age of Avani is 3 years, what is the
age of Rekha?
2. At present, the ratio between the ages of Amar and Norman is 4:3. After 6 years, Amar’s age will be 26
years. What is the age of Norman at present?
3. The ratio of the father’s age to the son’s age is 4:1 the product of their ages is 196. What will the ratio
of their ages after 5 years?
4. The ratio between the present ages of P and R is 5:3. The ratio between P’s age four years ago and R’s
age four years hence is 1:1. What is the ratio between P’s age 4 years hence and B’s age four years ago?
5. The ages of two persons differ by 16 years. 6 years ago, the elder one was 3 times as old as the
younger one. What are their present ages of the elder person?
6. Kamala got married 6 years ago. Today her age is (5/4) times her age at the time of marriage. Her son's
age is ( 1/10) times her age. Her son's age is ?
7. Pushpa is twice as old as Rita was two years ago. If the difference between their ages is 2 years, how
old is Pushpa today?
8. A person's present age is two-fifth of the age of his mother. After 8 years, he will be one-half of the
age of his mother. How old is the mother at present?
9. The age of Rani is 5 times the age of her daughter. After 12 years the age of Rani will be thrice the age
of her daughter. Find the present age of Rani’s daughter.
10. The ratio between the present ages of Ram and Shyam is 2:3. Four years ago the ratio between their
ages was 5:8. What will be Doom’s age after 7 years?
Partnership:
1. Partnership:
When two or more than two persons run a business jointly, they are called partners and the deal is
known as partnership.
2. Ratio of Divisions of Gains:
I. When investments of all the partners are for the same time, the gain or loss is
distributed among the partners in the ratio of their investments.
Suppose A and B invest Rs. x and Rs. y respectively for a year in a business, then at the end of the year:
(A's share of profit) : (B's share of profit) = x : y.
II. When investments are for different time periods, then equivalent capitals are calculated
for a unit of time by taking (capital x number of units of time). Now gain or loss is divided
in the ratio of these capitals.
Suppose A invests Rs. x for p months and B invests Rs. y for q months then,
(A's share of profit) : (B's share of profit)= xp : yq.
3. Working and Sleeping Partners:
A partner who manages the business is known as a working partner and the one who simply invests the
money is a sleeping partner.
Examples:
1. Raj invested Rs 76000 in a business. After few months Monty joined him and invests Rs 57000. At the
end of year both of them share the profits at the ratio of 2:1. After how many months Monty joined?

Solution - We can simply compute per month investment of both partnership

Raj invested Rs 76,000 for 12 months and Monty invested Rs 57,000 for x months.

Now 76000 × 12 / 57000 × x = 2 :1

⇒ 76 × 12 / 2 = 57x

⇒x=8

So Monty invested his money for 8 months and he joined after 4 months.
2. A and B started a business by investing money in ratio of 5:6. C joined them after few months by
sharing an amount equal to B's share. At the end of year 20% profit was earned which was equal equal
to Rs 98,000. How much money was invested by C ?
Solution: First of all we will calculate the weighted ratios

⇒ A = 5 × 12 = 60
⇒ B = 6 × 12 = 72
⇒ C = 6 × 6 = 36
Total investment at the end of year = 98000 × 100/20 = Rs 4,90,000
⇒ Investment by C = 490000 × 36 / 168 × 2 = Rs 210000
3. A, B and C shared profits in ratio of 5:7:8. They partnered for 14 months, 8 months and 7 months
respectively. What was he ratio of their investments?

Solution - Simply multiply profit sharing ratio with investment ratio to get investment amount ratio.

Let X is the total investment

⇒14 x = 5
⇒8x=7
⇒ 7x = 8

⇒ Final investment ratio = 20: 49: 64


5. Amar started a business investing Rs. 70,000. Ramki joined him after six months with an amount of Rs.
1, 05,000 and Sagar joined them with Rs. 1.4 lakhs after another six months. The amount of profit
earned should be distributed in what ratio among Aman, Rakhi and Sagar respectively, 3 years after
Aman started the business?
Solution:
Amar: Ramki: Sagar = (70000 X 36) : (105000 X 30) : (140000 X24) = 12 : 15 : 16.
6. A, B and C enter into a partnership. They invest Rs. 40,000, Rs. 80,000 and Rs. 1,20,000 respectively. At
the end of the first year, B withdrawn Rs. 40,000, while at the end of the second year, C withdraws Rs.
80,000. In what ratio will the profit be shared at the end of 3 years ?
Solution: A : B : C = (40,000 X 36) : (80,000 X 12 + 40,000 X 24) : (120,000 X 24 + 40,000 X 12) = 3: 4: 16
7. A invested Rs.2000/- in a firm which is half the investment of 'B's. But C's investment is sum of A and
B. Find their investments ratio:
Solution: Let Investment of A = Rs.2000/-
Let Investment of B = Rs.4000/- (2×Rs.2000)
Let Investment of C = Rs.6000/- (2000+4000)
A:B:C = 1:2:3
8. A, B and C jointly thought of engaging themselves in a business venture. It was agreed that A would
invest Rs. 6500 for 6 months, B, Rs. 8400 for 5 months and C, Rs. 10,000 for 3 months. A wants to be the
working member for which, he was to receive 5% of the profits. The profit earned was Rs. 7400.
Calculate the share of B in the profit?
Solution: For managing, A received = 5% of Rs. 7400 = Rs. 370.
Balance = Rs. (7400 - 370) = Rs. 7030.
Ratio of their investments = (6500 x 6) : (8400 x 5) : (10000 x 3)
= 39000: 42000: 30000
= 13: 14: 10
B's share = 7030 * (14/37)= Rs. 2660.
9 . A, B, C subscribe Rs. 50,000 for a business. A subscribes Rs. 4000 more than B and B Rs. 5000 more
than C. Out of a total profit of Rs. 35,000, A receives:
Solution: Let C = x.
Then, B = x + 5000 and A = x + 5000 + 4000 = x + 9000.
So, x + x + 5000 + x + 9000 = 50000
3x = 36000
x = 12000
A : B : C = 21000 : 17000 : 12000 = 21 : 17 : 12.
A's share = 35000 x (21/50) =14700.
10. A and B started a partnership business investing some amount in the ratio of 3 : 5. C joined then after
six months with an amount equal to that of B. In what proportion should the profit at the end of one
year be distributed among A, B and C?
Solution: Let the initial investments of A and B be 3x and 5x.
A : B : C = (3x x 12) : (5x x 12) : (5x x 6) = 36 : 60 : 30 = 6 : 10 : 5.
Exercise:
1. A,B and C enter into a partnership investing Rs 35000, Rs 45000 and 55000. Find the their respective
shares in annual profit of 40,500 ?
2 .A and B started a partnership business investing some amount in the ratio of 3 : 5. C joined them after
six months with an amount equal to that of B. In what proportion should the profit at the end of one
year be distributed amount A, B and C?
3 . A starts business with Rs. 3500 and after 5 months, B joins with A as his partner. After a year, the
profit is divided in the ratio 2:3. What is B's contribution in the capital?
4. Kamal started a business investing Rs 9000. After five months, Sameer joined with a capital of Rs 8000.
If at the end of the year, they earn a profit of Rs. 6970, then what will be the share of Sameer in the
profit?
5. Arun, Kiran and Vinay invested Rs. 8000, Rs. 4000 and Rs. 8000 respectively in a business. Arun left
after six months. If after eight months, there was a gain of Rs. 4005, then what will be the share of Kiran?
6. Shekhar started a business investing Rs. 25,000 in 1999. In 2000, he invested an additional amount of
Rs. 10,000 and Rajeev joined him with an amount of Rs. 35,000. In 2001, Shekhar invested another
additional amount of Rs. 10,000 and Jatin joined them with an amount of Rs. 35,000. What will be
Rajeev’s share in the profit of Rs. 1,50,000 earned at the end of 3 years from the start of the business in
1999?
7. A,B and C started a business with Rs.15000, Rs.25000 and Rs.35000 respectively. A was paid 10% of
the total profit as a salary and the balance was divided in the ration of investment. If A’s share is
Rs.4,200, then C’s share is?
8. A,B and C are three partners in a business. If twice the investment of A is equal to thrice the capital of
B and the capital of B is four times the capital of C. Out of a total profit of Rs.5940, the share of C is?
9. A, B and C enter into partnership by making investments in the ratio 3:5:7. After a year invests another
Rs.337600 while A withdraw Rs.45600. The ratio of investments then changes to 24:59:167. How much
did A invest initially?

10. A, B and C enter into a partnership in the ratio : : . After 4 months, A increases his share 50%. If
the total profit at the end of one year be Rs. 21,600, then B's share in the profit is?
Averages:
The term Average refers to the sum of all observations divided by the total number of observations.
Average is used quite regular in our day to day life. For example to calculate the average marks of the
students, Average height of a particular group etc. The term average is also referred to as ‘Mean’. Basic
formula to calculate the average is as follows:

1. The average of ages of 10 persons in a club was 32. What should be the age of the new person joining
in the club so as to increase the average by 4 ?
Solution: Total age of 10 persons = 10 x 32 = 320
Total age of 11 persons = 11 x 36 = 396 (as the new average is 4 more than persent average)
So the age of the person joining is = 396 - 320 = 76
Alternate method: If the age of the new person joining the club is 32 then there is no change in the
average. If the new average has to be 36, the person who is joining must contribute 4 years to all 11
persons. That is he must have an age 44 years above 32. So new average is 76.
2. The average weight of the teacher and six students is 12 kg which is reduced by 5 kg if the weight
of the teacher is excluded. How much does the teacher weigh?
Solution: Total age of the students and teacher together = 7 x 12 = 84
New average after excluding teacher = 6 x 7 = 42
Teachers weight = 74 - 42 = 42
Alternate method:
Teacher has taken her contribution of 5 kgs from each of the students. As she is contributing 30 kgs to
all students, once she is excluded those 30 kgs remains with her along with original weight 12kgs. So her
weight is 30 + 12 = 42 kgs
3. A batsman had a certain average of runs for 16 innings. In the 17th innings, he made a score of 87 runs
thereby increasing his average by 3. What is his average after 17 innings?
Solution: Assume his initial average = X
His total runs after 16 innings = 16X
After scoring 87 runs his average got increased by 3 to X + 3
So his total runs after 17 innings = 17 x (X+3)
But it was given that the difference in the total scores after 16 innings and 17 innings = 87
Therefore 17×(x+3)−16x=87⇒X=36
His new average = 36 + 3 = 39
Alternate method:
His 87 runs in the 17th innings contributed to all the seventeen innings to increase the average by 3. So
he must have scored 17 x 3 = 51 runs extra to maintain the average. So his previous average will be 87 -
51= 36. Present average = 39
4. In the first 10 overs of a cricket game, the run rate was only 3.2. What should be the run rate in the
remaining 40 overs to reach the target of 282 runs?
Solution: Runs scored in the first 10 overs = 10 × 3.2 = 32
Total runs = 282
Remaining runs to be scored = 282 - 32 = 250
Remaining overs = 40
Run rate needed = 250/40=6.25
5. A grocer has a sale of Rs. 6435, Rs. 6927, Rs. 6855, Rs. 7230 and Rs. 6562 for 5 consecutive months.
How much sale must he have in the sixth month so that he gets an average sale of Rs. 6500?
Solution: Let the sale in the sixth month =x

Then (6435+6927+6855+7230+6562+x)/6 =6500

=> 6435+6927+6855+7230+6562+x =6×6500

=> 34009+x=39000
=> x=39000−34009=4991
6. The captain of a cricket team of 11 members is 26 years old and the wicket keeper is 3 years older. If
the ages of these two are excluded, the average age of the remaining players is one year less than the
average age of the whole team. Find out the average age of the team.
Solution: Number of members in the team = 11

Let the average age of the team = x


=> (Sum of ages of all 11 members)/11=x

=> Sum of the ages of all 11 members = 11x

Age of the captain = 26


Age of the wicket keeper = 26 + 3 = 29

Sum of the ages of 9 members of the team excluding captain and wicket keeper
=11x−26−29=11x−55

Average age of 9 members of the team excluding captain and wicket keeper
=(11x−55)/9

Given that (11x−55)/9=(x−1)

⇒11x−55=9(x−1)⇒11x−55=9x−9⇒2x=46⇒x=462=23 years.
7. The average monthly income of A and B is Rs. 5050. The average monthly income of B and C is Rs.
6250 and the average monthly income of A and C is Rs. 5200. What is the monthly income of A?
Solution: Let monthly income of A = a
monthly income of B = b
monthly income of C = c

a + b = 2 × 5050.... (Equation 1)
b + c = 2 × 6250 .... (Equation 2)
a + c = 2 × 5200 .... (Equation 3)

(Equation 1) + (Equation 3) - (Equation 2)


=> a + b + a + c - (b + c) = (2 × 5050) + (2 × 5200) - (2 × 6250)
=> 2a = 2(5050 + 5200 - 6250)
=> a = 4000

i.e., Monthly income of A = 4000.


8. In Kiran's opinion, his weight is greater than 65 kg but less than 72 kg. His brother does not agree with
Kiran and he thinks that Kiran's weight is greater than 60 kg but less than 70 kg. His mother's view is that
his weight cannot be greater than 68 kg. If all are them are correct in their estimation, what is the
average of different probable weights of Kiran?
Solution: Let Kiran's weight = x. Then
According to Kiran, 65 < x < 72 ----(equation 1)
According to brother, 60 < x < 70 ----(equation 2)
According to mother, x ≤ 68 ----(equation 3)
Given that equation 1,equation 2 and equation 3 are correct. By combining these equations, we can
write as
65<x≤68
i.e., x = 66 or 67 or 68
Average of different probable weights of Kiran = (66+67+68)/3=67.
9. A student's mark was wrongly entered as 83 instead of 63. Due to that the average marks for the class
got increased by 0.5. What is the number of students in the class?
Solution: Let the total number of students = x
the average marks increased by 12 due to an increase of 83 - 63 = 20 marks.
But total increase in the marks = (½) ×x=x/2
Hence we can write as
(x/2)=20⇒x=20×2=40.
10. The average age of husband, wife and their child 3 years ago was 27 years and that of wife and the
child 5 years ago was 20 years. What is the present age of the husband?
Solution: Let the present age of the husband = h
Present age of the wife = w
Present age of the child = c

3 years ago, average age of husband, wife and their child = 27


=> Sum of age of husband, wife and their child before 3 years = 3 × 27 = 81
=> (h-3) + (w-3) + (c-3) = 81
=> h + w + c = 81 + 9 = 90 --- equation(1)

5 years ago, average age of wife and child = 20


=> Sum of age of wife and child before 5 years = 2 × 20 = 40
=> (w-5) + (c-5) = 40
=> w + c = 40 + 10 = 50 --- equation(2)

Substituting equation(2) in equation(1)


=> h + 50 = 90
=> h = 90 - 50 = 40

i.e., Present age of the husband = 40


Exercise:
1. There are two divisions A and B of a class, consisting of 36 and 44 students respectively. If the average
weight of divisions A is 40 kg and that of division b is 35 kg. What is the average weight of the whole
class?
2. A batsman makes a score of 87 runs in the 17th inning and thus increases his averages by 3. What is
his average after 17th inning?
3. The average age of boys in a class is 16 years and that of the girls is 15 years. What is the average age
for the whole class?
4. The average age of 36 students in a group is 14 years. When teacher's age is included to it, the average
increases by one. Find out the teacher's age in years?
5. The average of 11 numbers is 10.8. If the average of the first six be 10.4 and that of the last six is 11.5.
Then the middle (6th) number is?
6. Amithab's average expenditure for the January to June is Rs. 4200 and he spends Rs.1200 in January
and Rs.1500 in July. The average expenditure for the months of February to July is?
7. The mean of 16 items was found to be 30. On rechecking, it was found that two items were wrongly
taken as 22 and 18 instead of 32 and 28 respectively. Find the correct mean?
8. The average monthly salary of 20 employees in an organization is Rs. 1500. If the manager's salary is
added, then the average salary increases by Rs. 100. What is the manager's monthly salary?
9. The aggregate monthly expenditure of a family was $ 6240 during the first 3 months, $ 6780 during
the next 4 months and $ 7236 during the last 5 months of a year. If the total saving during the year is $
7080, find the average monthly income of the family.
10. The average salary of all the workers in a workshop is Rs. 8000. The average salary of 7 members is
Rs. 12000 and the average salary of the rest is Rs. 6000.The total number of workers in the workshop?
Mixture & Allegations:
1. Allegation:
It is the rule that enables us to find the ratio in which two or more ingredients at the given price must be
mixed to produce a mixture of desired price.
2. Mean Price:
The cost of a unit quantity of the mixture is called the mean price.
3. Rule of Allegation:
(Quantity of cheaper)/( Quantity of dearer)= (C.P. of dearer - Mean Price)/ (Mean price - C.P. of cheaper)
4. Suppose a container contains x of liquid from which y units are taken out and replaced by water.

After n operations, the quantity of pure liquid = [x (1- (y/x) N]


Examples:
1. The milk and water in two vessels A and B are in the ratio 4:3 and 2:3 respectively. In what ratio the
liquids in both the vessels be mixed to obtain a new mixture in vessel c consisting half milk and half
water?
Solution:
Milk in 1 litre mixture of A = 4/7 litre.
Milk in 1 litre mixture of B = 2/5 litre.
Milk in 1 litre mixture of C = 1/2 litre.
By rule of allegation we have required ratio X: Y
X : Y
4/7 2/5
\ /
(Mean ratio)
(1/2)
/ \
(1/2 – 2/5) : (4/7 – 1/2)
1/10 1/1 4

So Required ratio = X: Y = 1/10: 1/14 = 7:5


2. A vessel is filled with liquid, 3 parts of which are water and 5 parts of syrup. How much of the mixture
must be drawn off and replaced with water so that the mixture may be half water and half syrup?
Explanation:
Suppose the vessel initially contains 8 litres of liquid.
Let x litres of this liquid be replaced with water.
Quantity of water in new mixture = litres.
Quantity of syrup in new mixture = litres.

=> 5x + 24 = 40 - 5x
=> 10x = 16 => x = 8/5
So, part of the mixture replaced = = 1/5.
3. A container contains 40 litres of milk. From this container 4 litres of milk was taken out and replaced
by water. This process was repeated further two times. How much milk is now contained by the
container?
Explanation: Amount of milk left after 3 operations
litres.
4. A merchant has 1000 kg of sugar part of which he sells at 8% profit and the rest at 18% profit. He gains
14% on the whole. The Quantity sold at 18% profit is?
Explanation:
By the rule of allegation:
Profit of first part Profit of second part

So, ratio of 1st and 2nd parts = 4 : 6 = 2 : 3.


Quantity of 2nd kind = (3/5 x 1000) kg = 600 kg.
5. A can contains a mixture of two liquids A and B in the ratio 7 : 5. When 9 litres of mixture are drawn
off and the can is filled with B, the ratio of A and B becomes 7 : 9. How many litres of liquid A was
contained by the can initially?
Explanation:
Suppose the can initially contains 7x and 5x litres of mixtures A and B respectively
Quantity of A in mixture left = litres.
Quantity of B in mixture left = litres.

So, the can contained 21 litres of A.


6. How many kilograms of sugar costing Rs. 9 per kg must be mixed with 27 kg of sugar costing Rs. 7 per
Kg so that there may be a gain of 10 % by selling the mixture at Rs. 9.24 per Kg?
Explanation:
By the rule of allegation:

C.P. of 1 kg sugar of 1st kind C.P. of 1 kg sugar of 2nd kind


Rs. 9 Rs. 7

Rs. 8.40 (SP= 9.24 and gain 10%, so CP= 8.40)


1.40 0.60
Ratio of quantities of 1st and 2nd kind = 1.40: 0.60 = 7: 3.
Let x kg of sugar of 1st kind be mixed with 27 kg of 2nd kind.
Then, 7: 3 = x: 27 or x = (7 x 27 / 3) = 63 kg.
7. The cost of Type 1 rice is Rs. 15 per kg and Type 2 rice is Rs.20 per kg. If both Type 1 and Type 2 are
mixed in the ratio of 2 : 3, then the price per kg of the mixed variety of rice is?
Explanation:
Let the price of the mixed variety be Rs. x per kg. By the rule of allegation, we have :
Cost of 1 kg of type 1 rice Cost of 1 kg of type 2 rice

(20-x)/(x-15) = 2/3 60 - 3x = 2x - 30
 5x = 90 x = 18.
8. In a mixture of milk and water, there is only 26% water. After replacing the mixture with 7 liters of
pure milk, the percentage of milk in the mixture become 76%. The quantity of mixture is?
Explanation:
Milk Water
74% 26% (initially)
76% 24% (after replacement)
Left amount = Initial amount ((1- (replacement amount/total amount))
24 = 26(1- 7/k)
K= 91 liters.
Exercise:
1. Tea worth Rs. 126 per kg and Rs. 135 per kg are mixed with a third variety of tea in the ratio 1 : 1 : 2. If
the mixture is worth Rs. 153 per kg, what is the price of the third variety per kg?
2. Two vessels A and B contain spirit and water in the ratio 5 : 2 and 7 : 6 respectively. Find the ratio in
which these mixtures be mixed to obtain a new mixture in vessel C containing spirit and water in the
ratio 8: 5?
3. The cost of Type 1 material is Rs. 15 per kg and Type 2 material is Rs.20 per kg. If both Type 1 and Type
2 are mixed in the ratio of 2 : 3, then what is the price per kg of the mixed variety of material?
4. In what ratio should rice at Rs.9.30 per kg be mixed with rice at Rs. 10.80 per kg so that the mixture be
worth Rs.10 per kg?
5. In what ratio must tea worth Rs. 60 per kg be mixed with tea worth Rs. 65 a kg such that by selling the
mixture at Rs. 68.20 a kg , there can be a gain 10%?
6. In what ratio must water be mixed with milk costing Rs.12 per litre in order to get a mixture worth of
Rs.8 per liter?
7. A merchant has 1000 kg of sugar part of which he sells at 8% profit and the rest at 18% profit. He gains
14% on the whole. The quantity sold at 18% profit is?
8. How many liters’ of water must be added to 16 liters of milk and water containing 10% water to make
it 20% water in it?
9. We have a 630 ml mixture of milk and water in the ratio 7:2. How much water must be added to make
the ratio 7:3?
10. Rs.460 was divided among 41 boys and girls such that each boy got Rs.12 and each girl got Rs.8.
What is the number of boys?
Important Formulas - Time and Work
 If A can do a piece of work in n days, work done by A in 1 day = 1/n

 If A does 1/n work in a day, A can finish the work in n days

 If M1 men can do W1 work in D1 days working H1 hours per day and M2 men can do W2
work in D2 days working H2 hours per day (where all men work at the same rate), then
M1 D1 H1 / W1 = M2 D2 H2 / W2

 If A can do a piece of work in p days and B can do the same in q days, A and B together
can finish it in (pq / (p+q)) days

 If A is thrice as good as B in work, then


Ratio of work done by A and B = 3: 1
Ratio of time taken to finish a work by A and B = 1: 3
Pipes and Cisterns:
1. If a pipe can fill a tank in x hours, part filled in 1 hour = 1/x.
2. If a pipe can empty the tank in y hours, part emptied in 1 hour = 1/y.
3. Let a pipe can fill a tank in x hours and another pipe can empty the full tank in y hours. We
can examine two cases here.

Case 1: x < y
In this case, the net part filled in 1 hour = (1/x)–(1/y).
Case 2: y < x
In this case, the net part emptied in 1 hour = (1/y)–(1/x).
Examples:
1. A can do a piece of work in 10 days, B in 15 days. They work together for 5 days, the rest of
the work is finished by C in two more days. If they get Rs. 3000 as wages for the whole work,
what are the daily wages of A, B and C respectively (in Rs)?
Explanation:
A's 5 days work = 50%
B's 5 days work = 33.33%
C's 2 days work = 16.66% [100- (50+33.33)]
Ratio of contribution of work of A, B and C = =3:2:1

A's total share = Rs. 1500


B's total share = Rs. 1000
C's total share = Rs. 500
A's one day's earning = Rs.300
B's one day's earning = Rs.200
C's one day's earning = Rs.250
2. 12 men can complete a work in 8 days. 16 women can complete the same work in 12 days. 8
men and 8 women started working and worked for 6 days. How many more men are to be
added to complete the remaining work in 1 day?
Explanation:
1 man's 1 day work = ; 1 woman's 1 day work =
work done in 6 days=
Remaining work =
(8 men +8 women)'s 1 day work = =
Remaining work=
work is done in 1 day by 1 man
 work will be done in 1 day by men
3. A can do a certain work in the same time in which B and C together can do it.If A and B
together could do it in 20 days and C alone in 60 days ,then B alone could do it in?
Explanation:
(A+B)'s 1 day's work=1/20
C's 1 day work=1/60
(A+B+C)'s 1 day's work=
Also A's 1 day's work = (B+C)'s 1 day's work
we get: 2 * (A's 1 day 's work)=1/15
=>A's 1 day's work=1/30
B's 1 day's work=
So, B alone could do the work in 60 days.
4. A group of workers was put on a job. From the second day onwards, one worker was
withdrawn each day. The job was finished when the last worker was withdrawn. Had no worker
been withdrawn at any stage, the group would have finished the job in 55% of the time. How
many workers were there in the group?
Explanation:

=> n= 10
5. P can complete a work in 12 days working 8 hours a day. Q can complete the same work in 8
days working 10 hours a day. If both p and Q work together, working 8 hours a day, in how many
days can they complete the work?
Explanation:
P can complete the work in (12 * 8) hrs = 96 hrs
Q can complete the work in (8 * 10) hrs=80 hrs
P's 1 hour work=1/96 and Q's 1 hour work= 1/80
(P+Q)'s 1 hour's work = =
So both P and Q will finish the work in hrs
 Number of days of 8 hours each =
6. The ratio of efficiency of A is to C is 5:3. The ratio of number of days taken by B is to C is 2:3. A
takes 6 days less than C, when A and C completes the work individually. B and C started the
work and left after 2 days. The number of days taken by A to finish the remaining work is?
Explanation:
A : B : C
Efficiency 10 : 9 : 6
No of days 9x : 10x : 15x
15x-9x = 6
x=1
Number of days taken b A = 9
Number of days taken by B= 10
Number of days taken by C = 15
Work done by B and C in initial 2 days = =
Rest work =
Number of days required by A to finish work = = 6 days.
7. Two pipes A and B can fill a tank in 15 minutes and 20 minutes respectively. Both the pipes
are opened together but after 4 minutes, pipe A is turned off. What is the total time required to
fill the tank?
Explanation:
Part filled in 4 minutes =

Remaining part =

Part filled by B in 1 minute =

The tank will be full in (4 min. + 10 min. + 40 sec.) = 14 min. 40 sec.


8. A water tank is two-fifth full. Pipe A can fill a tank in 10 minutes and pipe B can empty it in 6
minutes. If both the pipes are open, how long will it take to empty or fill the tank completely?
Explanation:
Clearly, B is faster than pipe A and so, the tank will be emptied.
Part to be emptied = 2/5
Part emptied by (A+B) in 1 minute=

so, the tank will be emptied in 6 min


9. Two pipes can fill a tank in 20 and 24 minutes respectively and a waste pipe can empty 3
gallons per minute. All the three pipes working together can fill the tank in 15 minutes. The
capacity of the tank is?

Explanation:
Work done by the waste pipe in 1 minute = [-ve sign means emptying]
Volume of part = 3 gallons
Volume of whole = (3 x 40) gallons = 120 gallons
10. One pipe can fill a tank three times as fast as another pipe. If together the two pipes can fill
the tank in 36 min, then the slower alone will be able to fill the tank in?
Explanation:
Let the slower pipe alone fill the tank in x minutes.
Then, faster pipe will fill it in x/3 minutes.
(1/x) + (3/x) = 1/36
X=144 min.
Exercise:
1. A tap can fill a tank in 6 hours. After half the tank is filled then 3 more similar taps are
opened. What will be total time taken to fill the tank completely?
2. A water tank is two-fifth full. Pipe A can fill a tank in 10 minutes and pipe B can empty in 6
minutes. If both the pipes are open, how long will it take to empty or fill the tank completely?
3. Pipes A and B can fill a tank in 5 hours and 6 hours respectively. Pipe C can empty it in 12
hours. If all the three pipes are opened together, then the tank will be filled in?
4. A leak in the bottom of a tank can empty the full tank in 6 hours. An inlet pipe fills water at
the rate of 4 liters a minute. When the tank is full, the inlet is opened and due to the leak the
tank is empty in 8 hours. The capacity of the tank (in liters) is?
5. An electric pump can fill a tank in 3 hours. Because of a leak in the tank, it took 3 hours 30
min to fill the tank. In what time the leak can drain out all the water of the tank and will make
tank empty?
6. A tank can be filled by a tap in 20 minutes and by another tap in 60 minutes. Both the taps
are kept open for 10 minutes and then the first tap is shut off. After this, the tank will be
completely filled in what time?
7. A tank is filled by three pipes with uniform flow. The first two pipes operating simultaneously
fill the tank in the same time during which the tank is filled by the third pipe alone. The second
pipe fills the tank 5 hours faster than the first pipe and 4 hours slower than the third pipe. Find
the time required by the first pipe to fill the tank?
8. Two pipes A and B can fill a tank in 6 hours and 4 hours respectively. If they are opened on
alternate hours and if pipe A is opened first, in how many hours, the tank shall be full?
9. A tyre has two punctures. The first puncture alone would have made the tyre flat in 9 minutes
and the second alone would have done it in 6 minutes. If air leaks out at a constant rate, how
long does it take both the punctures together to make it flat?
10. To complete a work A and B takes 8 days, B and C takes 12 days, A,B and C takes 6 days. How
much time A and C will take?
11. A is thrice as good a workman as B and takes 10 days less to do a piece of work than B takes.
B alone can do the whole work in?
12. A can do a piece of work in 15 days and B alone can do it in 10 days. B works at it for 5 days
and then leaves. A alone can finish the remaining work in?
13. A completes 80% of a work in 20 days. Then B also joins and A and B together finish the
remaining work in 3 days. How long does it need for B if he alone completes the work?
14. 4 men and 6 women finish a job in 8 days, while 3 men and 7 women finish it in 10 days. In
how many days will 10 women working together finish it?
15. A piece of work can be done by 6 men and 5 women in 6 days or 3 men and 4 women in 10
days. It can be done by 9 men and 15 women in how many days?
Time & Distance:
Key Concepts:

1. Distance = Speed × Time


2. 1 km/hr = 5/18 m/s
3. If the ratios of speed is a: b : c, then the ratios of time taken is : 1/a:1/b:1/c.

Relation between variables:

To distance, Speed and Time both are directly proportional and To speed, Time is inversely
proportional. S∝D and T∝D i.e. if speed is doubled, distance covered in a given time also gets
doubled and, S∝1/T i.e. if speed is doubled, time taken to cover a distance will be half.

Average speed is defined as =.Total distance travelled/Total time taken


Relative speed :

If two bodies are moving (in the same direction or in the opposite direction), then the speed of
one body with respect to the other is called its relative speed.

Relative speed is a phenomenon that we observe every day. Suppose you are travelling in
college bus and there is a second bus coming in the opposite direction, then it seems that the
second Bus is moving much faster than actual. If both the Buses were moving in the same
direction at same speeds, they seem to be stationary if seen from one of these Busses, even
though they might actually be at a speed of 100 km/hr each. So what you actually observe is
your speed relative to the other.

Concepts:

1. If two objects are moving in opposite directions towards each other at speeds u and v, then
relative speed = Speed of first + Speed of second = u + v.
This is also the speed at which they are moving towards each other or the speed at which they
may be moving away from each other.

2. If the two objects move in the same direction with speeds u and v, then
relative speed = difference of their speeds = u – v.
This is also the speed at which the faster object is either drawing closer to the slower object or
moving away from the slower object as the case may be.

3. If the two objects start from A and B with speeds u and v respectively, and after crossing each
other take a and b hours to reach B and A respectively, then u: v = √a/b

Note: In case of Trains moving in the opposite directions or in the same direction, the total
distance required to be traveled before they cross each other completely is equal to the sum of
the lengths of the two trains. This distance is covered at the relative speeds of the trains.

Examples:
1. A man walking at the rate of 5 km/hr crosses a bridge in 15 minutes. The length of the bridge
(in meters) is?
Explanation:
Speed = (5x5/18) m/sec
= 25/18 m/sec.
Distance covered in 15 minutes = (25/18 x 15 x 60) m = 1250 m.
2. A man on tour travels first 160 km at 64 km/hr and the next 160 km at 80 km/hr. The average
speed for the first 320 km of the tour is?
Explanation:
Total time taken = (160/64 + 160/8) hrs = 9/2 hrs.
Average speed = (320 x 2/9) km.hr = 71.11 km/hr.
3. The distance of the college and home of Rajeev is 80km. One day he was late by 1 hour than
the normal time to leave for the college, so he increased his speed by 4km/h and thus he
reached to college at the normal time. What is the changed (or increased) speed of Rajeev?
Explanation: Let the normal speed be x km/h, then

x = 16 km/h
(x + 4) = 20 km/h
Therefore increased speed = 20 km/h.
4. A person goes to his office at 1/3rd of the speed at which he returns from his office. If the
average speed during the whole trip is 12 m /h. What is the speed of the person while he was
going to his office?
Explanation:
u = k, v= 3k

5. Akash leaves Mumbai at 6 am and reaches Bangalore at 10 am. Prakash leaves Bangalore at
8 am and reaches Mumbai at 11:30 am. At what time do they cross each other?
Explanation:
Time taken by Akash = 4 h
Time taken by Prakash = 3.5 h
For your convenience take the product of times taken by both as a distance.
Then the distance = 14km
Since, Akash covers half of the distance in 2 hours(i.e. at 8 am)
Now, the rest half (i.e. 7 km) will be covered by both prakash and akash
Time taken by them = 7/7.5 = 56 min
Thus, they will cross each other at 8: 56am.
6. Excluding stoppages, the speed of a bus is 54 kmph and including stoppages, it is 45 kmph.
For how many minutes does the bus stop per hour?
Explanation:
Due to stoppages, it covers 9 km less.

Time taken to cover 9 km = (9/54) hour


= (1/6)*60 minutes
= 10 minutes.
7. A thief is spotted by a policeman at a distance of 400 m. If the speed of the thief be 10 km/hr
and that of the policeman be 12 km/hr, at what distance will the policeman catch the thief?
Explanation:
Relative speed of the policeman = 2 km/hr. This means, every hour the distance between police
man and thief get reduces by 2000 meters. To cover 400 meters, Police needs 400/2000 Hrs or
1/5th hour. But in 1/5th hour police covers 12/5 km = 2.4 km.
8. P and Q are two towns. Ravi covers the distance from P to Q on cycle at 16 km/hr. However,
he covers the distance from Q to P on foot at 9 km/hr. His average speed during the whole
journey is?
Explanation: use the formula (2ab)/(a + b).
Here a, b are speeds.
Exercise:
1. A train is moving with a speed of 92.4 km/hr. How many metres will it cover in 10 minutes?
2. A boy goes to school with a speed of 3 km/hr and returns to the village with a speed of 2
km/hr. If he takes 5 hours in all, the distance between the village and the school is?
3. A car completes a certain journey in 8 hours. It covers half the distance at 40 km/hr. and the
rest at 60 km/hr. The length of the journey is?
4. A car covers four successive three km stretches at speeds of 10 km/hr, 30 km/hr and 60
km/hr respectively. Its average speed over this distance?
5. A car covers a distance in 50 minutes. If it runs at a speed of 48 km per hour on an average,
the speed at which the train must run to reduce the time of journey to 40 minutes will be?
6. Two cars start at the same time from Vijayawada and Madras and proceed towards each
other at 16 km/hr and 21 km/hr respectively. When they meet, it is found that one car has
travelled 60 km more than the other. The distance between the two stations is?
7. Rama travels a certain distance at 3 km/hr and reaches 15 min late. If he travels at 4 km/hr he
reaches 15 min earlier. The distance he has to travel is?
8. A man travels 35 km partly at 4 km/hr and at 5 km/hr. If he covers former distance at 5 km/hr
and later distance at 4 km/hr, he could cover 2 km more in the same time. The time taken to
cover the whole distance at original rate is?
9. A man walking at 3 km/hr crosses a square field diagonally in 2 min. The area of the field is?
10. The distance between two stations A and B is 220 km. A train leaves A towards B at an
average speed of 80 km/hr. After half a hour, another train leaves B towards A at an average
speed of 100 km/hr. The distance of the point where the two trains meet, from A is?
Problems on trains:
Key concepts:

1. If two objects are moving in opposite directions towards each other at speeds u and v, then
relative speed = Speed of first + Speed of second = u + v.
This is also the speed at which they are moving towards each other or the speed at which they
may be moving away from each other.

2. If the two objects move in the same direction with speeds u and v, then
relative speed = difference of their speeds = u – v.

This is also the speed at which the faster object is either drawing closer to the slower object or
moving away from the slower object as the case may be.
Important Models:

Model 1. 1 Pole and I Train:


Length of The Train (m) = Speed of the Train (m/s) × Time taken to cross the pole (s)

Model 2. 1 Train and 1 Platform:


Length of the Train + Length of the Platform (m) = Speed of the Train (m/s) × Time taken to cross
the platform (s)

Model 3. One Train with speed v1 and one moving person with speed v2
Case 1: If both are moving in same direction
Length of The Train (m) = [Speed of the Train - Speed of the Man] (m/s) × Time taken to cross
the man (s)

Case 2: If both are moving in opposite direction


Length of The Train (m) = [Speed of the Train + Speed of the Man] (m/s) × Time taken to cross
the man (s)

Model 4. 2 Trains with speeds v1,v2


Case 1: If both are moving in same direction
[Length of The Train 1 + Length of the Train 2](m) = [Speed of the Train1 - Speed of the Train 2]
(m/s) × Time taken to cross (s)

Case 2: If both are moving in opposite direction


[Length of The Train 1 + Length of the Train 2](m) = [Speed of the Train1 + Speed of the Train 2]
(m/s) × Time taken to cross (s).
Examples:
1. A man sitting in a train which is traveling at 50 kmph observes that a goods train, traveling in
opposite direction, takes 9 seconds to pass him. If the goods train is 280 m long, find its speed?
Explanation:
Relative speed = m / sec = kmph = 112 kmph.
Speed of goods train = (112 - 50) kmph = 62 kmph.
2. Two trains 140 m and 160 m long run at the speed of 60 km/hr and 40 km/hr respectively in
opposite directions on parallel tracks. The time (in seconds) which they take to cross each other,
is?
Explanation: Relative speed = (60 + 40) km/hr =[ 100 x ( 5 / 18 ) ]m/sec = ( 250 /9 ) m/sec.
Distance covered in crossing each other = (140 + 160) m = 300 m.
Required time = [ 300 x ( 9/250 ) ] sec = ( 54/ 5 )sec = 10.8 sec.
3. Two trains having equal lengths, take 10 seconds and 15 seconds respectively to cross a post.
If the length of each train is 120 meters, in what time (in seconds) will they cross each other
when traveling in opposite direction?
Explanation:
Speed of train 1 = = 12 m/sec
Speed of train 2 = = 8 m/sec
if they travel in opposite direction, relative speed = 12 + 8 = 20 m/sec

distance covered = 120 + 120 = 240 m

time = distance/speed = 240/20 = 12 sec


4. A jogger is running at 9 kmph alongside a railway track is 240 meters ahead of the engine of a
120 meters long train running at 45 kmph in the same direction. In how much time will the train
pass the jogger?
Explanation: Speed of the train relative to jogger = (45-9) km/hr = 36 km/hr

= = 10 m/sec
Distance to be covered =(240 + 120)m = 360 m

Time taken = sec = 36 sec.


5. A train of length 110 meter is running at a speed of 60 kmph. In what time, it will pass a man
who is running at 6 kmph in the direction opposite to that in which the train is going?
Explanation: Distance = 110 m

Relative speed = 60 + 6 = 66 kmph (Since both the train and the man are in moving in opposite
direction)

= m/sec = m/sec

Time taken to pass the man = = 6 s.


6. A 300 metre long train crosses a platform in 39 seconds while it crosses a signal pole in 18
seconds. What is the length of the platform?
Explanation:

Speed = m/sec = m/sec.


Let the length of the platform be x meters.
Then, = 3(x+300) = 1950 x = 350m.
7. A train is traveling at 48 kmph. It crosses another train having half of its length , traveling in
opposite direction at 42 kmph, in 12 seconds. It also passes a railway platform in 45 seconds.
What is the length of the platform?
Explanation:
Speed of train 1 = 48 kmph
Let the length of train 1 = 2x meter

Speed of train 2 = 42 kmph


Length of train 2 = x meter (because it is half of train 1's length)
Distance = 2x + x = 3x
Relative speed= 48+42 = 90 kmph = m/s = 25 m/s

Time = 12 s

Distance/time = speed

Length of the first train = 2x = 200 meter


Time taken to cross the platform= 45 s

Speed of train 1 = 48 kmph = 480/36 = 40/3 m/s

Distance = 200 + y [where y is the length of the platform]

8. A train overtakes two persons who are walking in the same direction to that of the train at 2 kmph
and 4 kmph and passes them completely in 9 and 10 seconds respectively. What is the length of the
train?
Explanation:
Let x is the length of the train in meter and v is its speed in kmph

x/9 = ( v-2)(10/36) ---(1)

x/10 =( v-4) (10/36) --- (2)

Dividing equation 1 with equation 2

10/9 = (v-2)/(v-4)

=> 10v - 40 = 9v - 18

=> v = 22

Substituting in equation 1, x/9 = 200/36 => x = 9×200/36 = 50 m.


Exercise:
1. A train having a length of 1/4 mile is traveling at a speed of 75 mph. It enters a tunnel 3 ½ miles long.
How long does it take the train to pass through the tunnel from the moment the front enters to the
moment the rear emerges?
2. Two trains, each 100 m long are moving in opposite directions. They cross each other in 8 seconds. If
one is moving twice as fast the other, the speed of the faster train is?
3. A train , having a length of 110 meter is running at a speed of 60 kmph. In what time, it will pass a man
who is running at 6 kmph in the direction opposite to that of the train?
4. A 300 metre long train crosses a platform in 39 seconds while it crosses a post in 18 seconds. What is
the length of the platform?
5. Two trains are running at 40 km/hr and 20 km/hr respectively in the same direction. If the fast train
completely passes a man sitting in the slower train in 5 seconds, the length of the fast train is?
6. A train 108 m long is moving at a speed of 50 km/hr . It crosses a train 112 m long coming from
opposite direction in 6 seconds. What is the speed of the second train?
7. How many seconds will a 500 meter long train moving with a speed of 63 km/hr, take to cross a man
walking with a speed of 3 km/hr in the direction of the train?
8. Two trains running in opposite directions cross a man standing on the platform in 27 seconds and 17
seconds respectively. If they cross each other in 23 seconds, what is the ratio of their speeds?
9. Two trains of equal length are running on parallel lines in the same direction at 46 km/hr and 36
km/hr. If the faster train passes the slower train in 36 seconds, what is the length of each train?
10. Two trains are moving in opposite directions with speed of 60 km/hr and 90 km/hr respectively. Their
lengths are 1.10 km and 0.9 km respectively. the slower train cross the faster train in seconds?

Boats & Streams:


All the basic concepts of Time speed and Distance apply to this chapter Boats and streams. We
have to adjust the speeds according to the point whether the boat is moving against the stream
or with the stream.

There are two parameters in these problems.

1. Speed of the Stream (V): This is the speed with which the river flows.
2. Speed of the boat in still water (U): If the river is still, this is the speed at which the boat
would be moving.

The effective speed of a boat in upstream = u – v.


The effective speed of a boat in downstream = u + v.
Examples:
1. A man can row upstream 10 kmph and downstream 20 kmph. Find the man rate in still water
and rate of the stream?
Explanation:
Please remember,
If a is rate downstream and b is rate upstream
Rate in still water = 1/2(a+b)
Rate of current = 1/2(a-b)

=> Rate in still water = 1/2(20+10) = 15 kmph


=> Rate of current = 1/2(20-10) = 5 kmph.
2. In one hour, a boat goes 11km along the stream and 5 km against it. Find the speed of the
boat in still water?
Explanation:
We know we can calculate it by 1/2(a+b)

=> 1/2(11+5) = 1/2(16) = 8 km/hr.


3. A man takes twice as long to row a distance against the stream as to row the same distance in favor of
the stream. The ratio of the speed of the boat in still water and stream is?
Explanation:
Let speed downstream = x kmph
Then Speed upstream = 2x kmph

So ratio will be,


(2x+x)/2 : (2x-x)/2
=> 3x/2 : x/2 => 3:1
4. A boat can travel with a speed of 16 km/hr in still water. If the rate of stream is 5 km/hr, then find the
time taken by the boat to cover distance of 84 km downstream?
Explanation:
It is very important to check, if the boat speed given is in still water or with water or against water.
Because, if we neglect it we will not reach on right answer. I just mentioned here because mostly
mistakes in this chapter are of this kind only.

Let’s see the question now.


Speed downstream = (16 + 5) = 21 kmph

Time = distance/speed = 84/21 = 4 hours.


5. A boat covers a certain distance downstream in 1 hour, while it comes back in 1 1⁄2 hours. If the speed
of the stream be 3 kmph, what is the speed of the boat in still water?
Explanation:
Let the speed of the water in still water =x
Given that speed of the stream = 3 kmph
Speed downstream = (x+3) kmph
Speed upstream = (x−3) kmph

He travels a certain distance downstream in 1 hour and come back in 1 1⁄2 hour.
i.e., distance travelled downstream in 1 hour = distance travelled upstream in 1 1⁄2 hour

Since distance = speed × time, we have


(x+3) × 1= (x−3)3/2
⇒2(x+3) = 3(x−3) ⇒2x+6=3x−9 ⇒x=6+9=15 kmph.
6. A man can row at 5 kmph in still water. If the velocity of current is 1 kmph and it takes him 1 hour to
row to a place and come back, how far is the place?
Explanation:
Speed in still water = 5 kmph
Speed of the current = 1 kmph

Speed downstream = (5+1) = 6 kmph


Speed upstream = (5-1) = 4 kmph

Let the required distance be x km


Total time taken = 1 hour

⇒(x/6) + (x/4) = 1 ⇒2x+3x=12 ⇒5x=12 ⇒x=2.4 km.


7. In a river flowing at 2 km/hr, a boat travels 32 km upstream and then returns downstream to the
starting point. If its speed in still water be 6 km/hr, find the total journey time?
Explanation:
Speed of the boat = 6 km/hr

Speed downstream = (6+2) = 8 km/hr


Speed upstream = (6-2) = 4 km/hr

Distance travelled downstream = Distance travelled upstream = 32 km

Total time taken


= Time taken downstream + Time taken upstream
=32/8+32/4 = 12 hr.
8. A man can row 40 kmph in still water and the river is running at 10 kmph. If the man takes 1 hr to row
to a place and back, how far is the place?
Explanation:
Let the distance be x

Speed upstream = (40 - 10) = 30 kmph


Speed downstream = (40 + 10) = 50 kmph

Total time taken = 1 hr


⇒(x/50)+( x/30) =1⇒8x/150=1
⇒x=150/8= 18.75 km
Exercise:

1. A girl can swim 3 km/hr in still water. If the velocity of the stream be 2 km/hr the time taken by her to
swim to a place 10 km upstream and back is?
2. A girl can row three quarters of a kilometer against the stream in 11(1/4) minutes and return in 7.5
minutes. The speed of the girl in still water is?
3. The current of a stream runs at 1km/hr. A motor boat goes 35 km. upstream and back again to the
starting point in 12 hours. The speed of motor boat in still water is?
4. A motor covers 24 km upstream and 36 km downstream in 6 hours, while it covers 36 km upstream
and 24km downstream in 612 hours. The velocity of the stream is?
5. A man rows to a place 48 km distant and back in 14 hours. He finds that he can row 4 km with the
stream in the same time as 3 km against the stream. The rate of the stream is?
6. Speed of a boat in standing water is 6 km/hr and the speed of the stream is 1.5 km/hr. A man rows to
a place at a distance of 22.5 km and comes back to the starting point. The total time taken by him, is?
7. A motor goes 40 km upstream in 8 hours and 36 km downstream in 6 hours. The speed of the boat in
standing water is?
8. A boat travels upstream from Q to P and downstream from P to Q in 3 hours. If the speed of the boat
in still water is 9 km/hr and the speed of the current is 3 km/hr, the distance between P and Q is?
1. Factorial Notation:
Let ‘n’ be a positive integer. Then, factorial n, denoted ‘n!’ is defined as:
n! = n (n - 1)(n - 2) ... 3.2.1.
Examples:
5! = (5 x 4 x 3 x 2 x 1) = 120.
4! = (4 x 3 x 2 x 1) = 24.
2. Permutations:
The different arrangements of a given number of things by taking some or all at a time are called
permutations.
Examples:
i. All permutations (or arrangements) made with the letters a, b, c by taking two at a time are (ab,
ba, ac, ca, bc, cb).
ii. All permutations made with the letters a, b, c taking all at a time are:
( abc, acb, bac, bca, cab, cba)
3. Number of Permutations:
Number of all permutations of n things, taken r at a time, is given by:

n
Pr = n(n - 1)(n - 2) ... (n - r + 1) = {n!/ (n-r)!}
Examples:
6
i. P2 = (6 x 5) = 30.
7
ii. P3 = (7 x 6 x 5) = 210.
4. Permutations under Some Conditions:
If there are n subjects of which p1 are alike of one kind; p2 are alike of another kind; p3 are alike
of third kind and so on and pr are alike of rth kind,
such that (p1 + p2 + ... pr) = n.
Then, the number of permutations of these n things = {n! /( p1! * p2! * ….. pr!)}
Number of permutations of n different things, taken r at a time, when a particular thing is to be always
included in each arrangement is: r (n-1) P (r−1).
Number of permutations of n different things, taken r at a time, when a particular thing is never taken in
each arrangement is: (n−1)Pr
Number of permutations of n different things, taken all at a time, when m specified things always come
together is: m! × (n−m+1)!
Number of permutations of n different things, taken all at a time, when m specified never come together
is: n! − [m!×(n−m+1)!]
The number of permutations of n dissimilar things taken r at a time when k(< r) particular things always
occur is: [(n−k)P(r−k)]×[rPk]
The number of permutations of n dissimilar things taken r at a time when k particular things never occur
is: (n−k)Pr
The number of permutations of n dissimilar things taken r at a time when repetition of things is allowed
any number of times is: nr
The number of permutations of n different things, taken not more than r at a time, when each thing may
occur any number of times is: n+n2+n3+....+nr=n(nr−1)/(n−1)
The number of permutations of n different things taken not more than r at a time:
nP1+nP2+nP3+...+nPr

5. Combinations:
Each of the different groups or selections which can be formed by taking some or all of a
number of objects is called a combination.
Examples:
1. Suppose we want to select two out of three boys A, B, C. Then, possible selections are
AB, BC and CA.
Note: AB and BA represent the same selection.
2. All the combinations formed by a, b, c taking ab, bc, ca.
3. The only combination that can be formed of three letters a, b, c taken all at a time is abc.
4. Various groups of 2 out of four persons A, B, C, D are:
AB, AC, AD, BC, BD, CD.
5. Note that ab, ba are two different permutations but they represent the same
combination.
6. Number of Combinations:
The number of all combinations of n things, taken r at a time is
n
Cr = { n!/( r!(n-r)!)}
Note:
n
i. Cn = 1 and nC0 = 1.
n
ii. Cr = nC(n - r)
11
C4 = (11 x 10 x 9 x 8)/ (1 x 2 x 3 x 4) = 330.
Examples:
1.In how many different ways can the letters of the word 'LEADING' be arranged in such a way that the
vowels always come together?
Explanation:
The word 'LEADING' has 7 different letters.
When the vowels EAI are always together, they can be supposed to form one letter.
Then, we have to arrange the letters LNDG (EAI).
Now, 5 (4 + 1 = 5) letters can be arranged in 5! = 120 ways.
The vowels (EAI) can be arranged among themselves in 3! = 6 ways.
Required number of ways = (120 x 6) = 720.
2. In how many ways can the letters of the word 'LEADER' be arranged?
Explanation:
The word 'LEADER' contains 6 letters, namely 1L, 2E, 1A, 1D and 1R.
Required number of ways = 6! / 2!
3. How many integers, greater than 999 but not greater than 4000, can be formed with the
digits 0, 1, 2, 3 and 4, if repetition of digits is allowed?
Explanation: The smallest number in the series is 1000, a 4-digit number.
The largest number in the series is 4000, the only 4-digit number to start with 4.
The left most digit (thousands place) of each of the 4 digit numbers other than 4000 can take
one of the 3 values 1 or 2 or 3.
Next 3 digits (hundreds, tens and units place) can take any of the 5 values 0 or 1 or 2 or 3 or 4.
Hence, there is 3 x 5 x 5 x 5 or 375 numbers from 1000 to 3999.
Including 4000, there will be 376 such numbers.
4. Suppose you can travel from a place A to a place B by 3 buses, from place B to place C by 4 buses,
from place C to place D by 2 buses and from place D to place E by 3 buses. In how many ways can you
travel from A to E?
Solution: The bus from A to B can be selected in 3 ways.
The bus from B to C can be selected in 4 ways.
The bus from C to D can be selected in 2 ways.
The bus from D to E can be selected in 3 ways.
So, by the General Counting Principle, one can travel from A to E in 3×4×2×3 = 72 ways.
5. If you have 6 New Year greeting cards and you want to send them to 4 of your friends, in how
many ways can this be done?
Solution:
We have to find number of permutations of 4 objects out of 6 objects.
This number is 6P4=6(6−1)(6−2)(6−3)=6×5×4×3=360
Therefore, cards can be sent in 360 ways.
6. Suppose 7 students are staying in a hall in a hostel and they are
Allotted 7 beds. Among them, Parvin does not want a bed next to Anju because Anju snores.
Then, in how many ways can you allot the beds?
Solution:
Let the beds be numbered 1 to 7.
Case 1:
Suppose Anju is allotted bed number 1.
Then, Parvin cannot be allotted bed number 2.
So Parvin can be allotted a bed in 5 ways.
After allotting a bed to Parvin, the remaining 5 students can be allotted beds in 5! ways.
So, in this case the beds can be allotted in 5×5! =600 ways.
Case 2:
Anju is allotted bed number 7.
Then, Parvin cannot be allotted bed number 6
As in Case 1, the beds can be allotted in 600 ways.
Case 3:
Anju is allotted one of the beds numbered 2,3,4,5or 6
Parvin cannot be allotted the beds on the right hand side and left hand side of Anju's bed.
For example, if Anju is allotted bed number 2, beds numbered 1 or 3 cannot be allotted to
Parvin.
Therefore, Parvin can be allotted a bed in 4 ways in all these cases.
After allotting a bed to Parvin, the other 5 can be allotted a bed in 5! ways.
Therefore, in each of these cases, the beds can be allotted 4×5! = 480ways.
=> The beds can be allotted in:
2×600+5×480=1200+2400 = 3600 ways.

7. In how many ways can an animal trainer arrange 5 lions and 4 tigers in a row so that no two lions are
together?
Solution:
They have to be arranged in the following way:
|L|T|L|T|L|T|L|T|L|
The 5 lions should be arranged in the 5 places marked 'L'.
This can be done in 5! Ways.
The 4 tigers should be in the 4 places marked 'T'.
This can be done in 4! Ways.
Therefore, the lions and the tigers can be arranged in 5! × 4! = 2880 ways.
8. How many arrangements of the letters of the word ‘BENGALI’ can be made?
(i) If the vowels are never together.
(ii) If the vowels are to occupy only odd places.
Solution:
There are 7 letters in the word ‘Bengali; of these 3 are vowels and 4 consonants.
(i) Considering vowels a, e, i as one letter, we can arrange 4+1 letters in 5! Ways in each of which
vowels are together. These 3 vowels can be arranged among themselves in 3! Ways.
=> Total number of words =5! × 3! =120×6=720
So there are total of 720 ways in which vowels are ALWAYS TOGEGHER.
Now, Since there are no repeated letters, the total number of ways in which the letters of the
word ‘BENGALI’ cab be arranged =7! = 5040
So, Total no. of arrangements in which vowels are never together:
=ALL the arrangements possible - arrangements in which vowels are ALWAYS TOGETHER
=5040−720=4320
(ii) There are 4 odd places and 3 even places. 3 vowels can occupy 4 odd places in 4P3 ways and
4 constants can be arranged in 4P4 ways.
Number of words =4P3 × 4P4 = 576
9. 12 points lie on a circle. How many cyclic quadrilaterals can be drawn by using these points?
Solution: For any set of 4 points we get a cyclic quadrilateral. Number of ways of choosing 4
points out of 12 points is 12C4=495
Therefore, we can draw 495 quadrilaterals.
10. In a box, there are 5 black pens, 3 white pens and 4 red pens. In how many ways can 2 black
pens, 2 white pens and 2 red pens can be chosen?
Solution:
Number of ways of choosing 2 black pens from 5 black pens 5C2= (5 x 4)/(1 x 2)=10
Number of ways of choosing 2 white pens from 3 white pens= 3C2 =3
Number of ways of choosing 2 red pens from 4 red pens = 4C2 = 6
=> By the Counting Principle, 2 black pens, 2 white pens, and 2 red pens can be chosen in
10×3×6 = 180ways.
11. A question paper consists of 10 questions divided into two parts A and B. Each part contains
five questions. A candidate is required to attempt six questions in all of which at least 2 should
be from part A and at least 2 from part B. In how many ways can the candidate select the
questions if he can answer all questions equally well?
Solution:
The candidate has to select six questions in all of which at least two should be from Part A and
two should be from Part B. He can select questions in any of the following ways:
Part
A Part B
(i) 2
4

(ii) 3
3

(iii) 4
2
If the candidate follows choice (i), the number of ways in which he can do so is:
5C2×5C4=10×5=50
If the candidate follows choice (ii), the number of ways in which he can do so is:
5C3×5C3=10×10=100
Similarly, if the candidate follows choice (iii), then the number of ways in which he can do so is:
5C4×5C2=5×10=50
Therefore, the candidate can select the question in 50+100+50 = 200 ways.
12. A committee of 5 persons is to be formed from 6 men and 4 women. In how many ways can this be
done when?
(i) At least 2 women are included?
(ii) At most 2 women are included?
Solution:
(i) When at least 2 women are included.
The committee may consist of
3 women, 2 men: It can be done in 4C3×6C2 ways.
Or, 4 women, 1 man: It can be done in 4C4×6C1 ways
Or, 2 women, 3 men: It can be done in 4C2×6C3 ways
Total number of ways of forming the committee:
4C3×6C2+4C4×6C1+4C2×6C3 = 186 ways
(ii) When at most 2 women are included
The committee may consist of
2 women, 3 men: It can be done in 4C2×6C3 ways
Or, 1 woman, 4 men: It can be done in 4C1×6C4 ways
Or, 100 % men: It can be done in 6C5 ways
=> Total number of ways of forming the committee:
4C2×6C3+4C1×6C4+6C5 = 186 ways.
Problems Involving Both Permutations and Combinations:
13. There are 5 novels and 4 biographies. In how many ways can 4 novels and 2 biographies can be
arranged on a shelf?
Solution:
4 novels can be selected out of 5 in 5C4 ways.
2 biographies can be selected out of4 in 4C2 ways.
Number of ways of arranging novels and biographies = 5C4×4C2=30
After selecting any 6 books (4 novels and 2 biographies) in one of the 30 ways, they can be
arranged on the shelf in 6! =720 ways.
By the Counting Principle, the total number of arrangements =30×720 = 21600.
14. From 5 consonants and 4 vowels, how many words can be formed using 3 consonants and 2 vowels?
Solution:
From 5 consonants, 3 consonants can be selected in 5C3 ways.
From 4 vowels, 2 vowels can be selected in 4C2 ways.
Now with every selection, number of ways of arranging 5 letters is 5P5
Total number of words = 5C3×4C2×5P5= 7200 ways.
Exercise:
1. How many ways can 4 prizes be given away to 3 boys (one boy can receive 1 prize), if each boy is
eligible for all the prizes?
2. There are fourteen juniors and twenty-three seniors in the Service Club. The club is to send four
representatives to the State Conference.
(i)- How many different ways are there to select a group of four students to attend the conference?
(ii)- If the members of the club decide to send two juniors and two seniors, how many different
groupings are possible?
3. A class photograph has to be taken. The front row consists of 6 girls who are sitting. 20 boys
are standing behind. The two corner positions are reserved for the 2 tallest boys. In how many
ways can the students be arranged?
4. If 5 × (nP3) = 4 × [(n+1)P3] find n?
5. How many words can be formed by re-arranging the letters of the word ASCENT such that A
and T occupy the first and last position respectively?
6. In how many ways can 5 different toys be packed in 3 identical boxes such that no box is
empty, if any of the boxes may hold all of the toys?
7. When six fair coins are tossed simultaneously, in how many of the outcomes will at most
three of the coins turn up as heads?
8. A college has 10 basketball players. A 5-member team and a captain will be selected out of
these 10 players. How many different selections can be made?
9. What is the total number of ways in which kohli can distribute 9 distinct gifts among his 8 distinct
girlfriends such that each of them gets at least one gift?
10. In how many ways can 15 people be seated around two round tables with seating capacities
of 7 and 8 people?
Probability:
1. Concept of Probability
Probability deals with the analysis of random phenomena. It is a way of assigning every event a
value between zero and one, with the requirement that the event made up of all possible
results is assigned a value of one.
2. Experiment
An operation which results in some well-defined outcomes is called an experiment.
2.1. Random Experiment
An experiment whose outcome cannot be predicted with certainty is called a random
experiment. In other words, if an experiment is performed many times under similar conditions
and the outcome of each time is not the same, then this experiment is called a random
experiment.
Example:
A). Tossing of a fair coin
B). Throwing of an unbiased die
C). Drawing of a card from a well shuffled pack of 52 playing cards.
3. Sample Space
The set of all possible outcomes of a random experiment is called the sample space for that
experiment. It is usually denoted by S.
Example:
A). When a die is thrown, any one of the numbers 1, 2, 3, 4, 5, 6 can come up.
Therefore, sample space:
S = {1, 2, 3, 4, 5, 6}
B). When a coin is tossed either a head or tail will come up, then the sample space w.r.to. the
tossing of the coin is:
S = {H, T}
C). When two coins are tossed, then the sample space is
3.1 Sample Point or Event Point
Each element of the sample spaces is called a sample point or an event point.
Example:
When a die is thrown, the sample space is S = {1, 2, 3, 4, 5, 6} where 1, 2, 3, 4, 5 and 6 are the
sample points.
3.2 Discrete Sample Space
A sample space S is called a discrete sample if S is a finite set.
4. Event
A subset of the sample space is called an event.
4.1. Problem of Events
Sample space S plays the same role as universal set for all problems related to the particular
experiment.
(i). ϕ is also the subset of S and is an impossible Event.
(ii). S is also a subset of S which is called a sure event or a certain event.
5. Types of Events
A. Simple Event or Elementary Event
An event is called a Simple Event if it is a singleton subset of the sample space S.
Example:
A). When a coin is tossed, then the sample space is
S = {H, T}
Then A = {H} occurrence of head and
B = {T} occurrence of tail are called Simple events.
B). When two coins are tossed, then the sample space is
S = {(H,H); (H,T); (T,H); (T,T)}
Then A = {(H,T)} is the occurrence of head on 1st
and tail on 2nd is called a Simple event.
B. Mixed Event or Compound Event or Composite Event
A subset of the sample space S which contains more than one element is called a mixed event
or when two or more events occur together; their joint occurrence is called a Compound Event.
Example:
When a dice is thrown, then the sample space is
S = {1, 2, 3, 4, 5, 6}
Then let A = {2, 4 6} is the event of occurrence of even and B = {1, 2, 4} is the event of
occurrence of exponent of 2 are Mixed events.
Compound events are of two types:
(i). Independent Events, and
(ii). Dependent Events
C. Equally Likely Events
Outcomes are said to be equally likely when we have no reason to believe that one is more
likely to occur than the other.
Example:
When an unbiased die is thrown all the six faces 1, 2, 3, 4, 5, 6 are equally likely to come up.
D. Exhaustive Events
A set of events is said to be exhaustive if one of them must necessarily happen every time the
experiments is performed.
Example:
When a die is thrown events 1, 2, 3, 4, 5, 6 form an exhaustive set of events.
Important:
We can say that the total number of elementary events of a random experiment is called the
exhaustive number of cases.
E. Mutually Exclusive Events
Two or more events are said to be mutually exclusive if one of them occurs, others cannot
occur. Thus if two or more events are said to be mutually exclusive, if not two of them can occur
together.
Hence, A1,A2,A3,...,An
are mutually exclusive if and only if Ai∩Aj=ϕ, for i≠j
Example:
A). When a coin is tossed the event of occurrence of a head and the event of occurrence of a
tail are mutually exclusive events because we cannot have both head and tail at the same time.
B). When a die is thrown, the sample space is S = {1, 2, 3, 4, 5, 6}
Let A is an event of occurrence of number greater than 4 i.e., {5, 6}
B is an event of occurrence of an odd number {1, 3, 5}
C is an event of occurrence of an even number {2, 4, 6}
Here, events B and C are Mutually Exclusive but the event A and B or A and C are not mutually
Exclusive.
F. Independent Events or Mutually Independent Events
Two or more event are said to be independent if occurrence or non-occurrence of any of them
does not affect the probability of occurrence of or non-occurrence of their events.
Thus, two or more events are said to be independent if occurrence or non-occurrence of any of
them does not influence the occurrence or non-occurrence of the other events.
Example:
Let bag contains 3 Red and 2 Black balls. Two balls are drawn one by one with replacement.
Let A is the event of occurrence of a red ball in first draw.
B is the event of occurrence of a black ball in second draw.
Then probability of occurrence of B has not been affected if A occurs before B. As the ball has
been replaced in the bag and once again we have to select one ball out of 5(3R + 2B) given balls
for event B.
6. Occurrence of an Event
For a random experiment, let E be an event
Let E = {a, b, c}. If the outcome of the experiment is either a or b or c then we say the event has
occurred.
Sample Space: The outcomes of any type
Event: The outcomes of particular type
6.1. Probability of Occurrence of an Event
Let S be the same space, then the probability of occurrence of an event E is denoted by P(E)
and is defined as
P(E)=n(E)/n(S) = (number of elements in E)/( number of elements in S)
P(E)=(number of favorable/particular cases)/( total number of cases)
Example:
A). When a coin is tossed, then the sample space is S = {H, T}
Let E is the event of occurrence of a head
E = {H}
B). When a die is tossed, sample space S = {1, 2, 3, 4, 5, 6}
Let A is an event of occurrence of an odd number
And B is an event of occurrence of a number greater than 4
A = {1, 3, 5} and B = {5, 6}
P(A)=Probability of occurrence of an odd number
=n(A)/n(S)
=3/6=12
P(B)=Probability of occurrence of a number greater than 4
=n(B)/n(S)
=2/6=1/3
7. Basic Axioms of Probability
Let S
Denote the sample space of a random experiment.
1. For any event E, P(E)≥0
2. P(S) =1
8. Some Basic Probability Theorems:
1. Let E' be the complement of E defined by E′=S−E
The following always holds: P(E)=1−P(E′)
2. If A⊆B, then P(A)≤P(B)
3. For any event E, 0≤ P(E) ≤1
4. For A and B, P(A∪B)=P(A)+P(B)−P(A∩B)
5. P(A∪B∪C)=P(A)+P(B)+P(C)−P(AB)−P(AC)−P(BC)+P(ABC)
6. Definition - Conditional probability:
P(A|B)=P(A∩B)/P(B) provided P(B)≠0
7. P(A∩B∩C)=P(A)∗P(B|A)∗P(C|A∩B)
8. If A1, A2, A3...... An are mutually independent events, then
P(A1∩A2∩...∩An) = P(A1)∗P(A2)∗...∗P(An)
9. Bayes' Theorem:
P(A|B)={P(B|A)∗P(A)} / {P(B|A)∗P(A)+P(B|A′)∗P(A′)}
Examples:
1. A number X is chosen at random from the numbers -3, -2, -1, 0, 1, 2, 3. What is the
probability that |X|<2?
Solution: |X| can take 7 values.
To get |X|<2 ( i.e., −2<X<+2) take X={−1,0,1}
⇒ P(|X|<2)= Favorable Cases/Total Cases

= 3/7
2. An urn contains 6 red, 5 blue and 2 green marbles. If 2 marbles are picked at random, what is
the probability that both are red?
Solution: P(Both are red),
=6C2/13C2
=5/26
3. A bag contains 12 white and 18 black balls. Two balls are drawn in succession without
replacement. What is the probability that first is white and second is black?
Solution:
The probability that first ball is white:
=12C1/30C1
=12/30
=2/5
Since, the ball is not replaced; hence the number of balls left in bag is 29.
Hence, the probability the second ball is black:
=18C1/29C1
=18/29
Required probability,
= (2/5) × (18/29)
=36/145.
4. A bag contains 6 white and 4 black balls .2 balls are drawn at random. Find the probability
that they are of same color?
Solution:
Let S be the sample space
Then n(S) = no of ways of drawing 2 balls out of (6+4) = = =45
Let E = event of getting both balls of same color
Then, n(E) = no of ways (2 balls out of six) or (2 balls out of 4)
= = = 15+6 = 21
Therefore, P(E) = n(E)/n(S) = 21/45 = 7/15.
5. A man and his wife appear in an interview for two vacancies in the same post. The probability
of husband's selection is (1/7) and the probability of wife's selection is (1/5). What is the
probability that only one of them is selected?
Solution:

6. Two cards are drawn at random from a pack of 52 cards. What is the probability that either
both are black or both are queen?
Solution: We have n(s) = = = 1326.
Let A = event of getting both black cards
B = event of getting both queens
A∩B = event of getting queen of black cards
n(A) = = = 325, n(B)= = = 6 and n(A∩B) = =1
P(A) = n(A)/n(S) = 325/1326;
P(B) = n(B)/n(S) = 6/1326 and
P(A∩B) = n(A∩B)/n(S) = 1/1326
P(A∪B) = P(A) + P(B) - P(A∩B) = (325+6-1) / 1326 = 330/1326 = 55/221
7. Two dice are tossed. The probability that the total score is a prime number is?
Solution: Clearly, n(S) = (6 x 6) = 36.
Let E = Event that the sum is a prime number.
Then E= { (1, 1), (1, 2), (1, 4), (1, 6), (2, 1), (2, 3), (2, 5), (3, 2), (3, 4), (4, 1), (4,3),(5, 2), (5, 6), (6,
1), (6, 5) }
n(E) = 15.
P(E) = n(E)/n(S) = 15/36 = 5/12.
8. In a class, 30% of the students offered English, 20% offered Hindi and 10% offered both. If a
student is selected at random, what is the probability that he. has offered English or Hindi ?
Solution:
9. A speaks truth in 75% of cases and B in 80% of cases. In what percentage of cases are they
likely to contradict each other, narrating the same incident?
Solution: Let A = Event that A speaks the truth
B = Event that B speaks the truth

Then P(A) = 75/100 = 3/4


P(B) = 80/100 = 4/5
P(A-lie) = = 1/4
P(B-lie) = = 1/5
Now, A and B contradict each other =[A true and B lies] or [A lies and B true]
= P(A).P(B-lie) + P(A-lie).P(B)
=
= = 35%
10. A word consists of 9 letters; 5 consonants and 4 vowels. Three letters are chosen at random.
What is the probability that more than one vowel will be selected?
Solution: 3 letters can be chosen out of 9 letters in ways.
More than one vowels ( 2 vowels + 1 consonant or 3 vowels ) can be chosen in
ways

Hence, required probability = = 17/42.


11. Out of 17 applicants 8 boys and 9 girls. Two persons are to be selected for the job. Find the
probability that at least one of the selected persons will be a girl?
Solution: The events of selection of two person is redefined as first is a girl and second is a boy or first is
boy and second is a girl or first is a girl and second is a girl.

So the required probability:


=

=
12. Two cards are drawn from a pack of well shuffled cards. Find the probability that one is a
club and other in King?
Solution: Let X be the event that cards are in a club which is not king and other is the king of
club.
Let Y be the event that one is any club card and other is a non-club king.
Hence, required probability:
=P(A)+P(B)
=

= = =
Exercise:
1. A bag contains 6 black and 8 white balls. One ball is drawn at random. What is the probability that the
ball drawn is white?
2. An unbiased die is tossed. Find the probability of getting a multiple of 3?
3. A bag contains 7 green and 5 black balls. Three balls are drawn one after the other. The probability of
all three balls being green, if the balls drawn are not replaced will be?
4. In a charity show tickets numbered consecutively from 101 through 350 are placed in a box. What is
the probability that a ticket selected at random (blindly) will have a number with a hundredth digit of 2?
5. A box contains 100 balls, numbered from 1 to 100. If three balls are selected at random and with
replacement from the box, what is the probability that the sum of the three numbers on the balls
selected from the box will be odd?
6. The probability of success of three students X, Y and Z in the one examination are 1/5, 1/4 and 1/3
respectively. Find the probability of success of at least two?
7. A box contains 10 bulbs, of which just three are defective. If a random sample of five bulbs is drawn,
find the probability that the sample contains exactly one defective bulb?
8. The letters of the word CASTIGATION is arranged in different ways randomly. What is the chance that
vowels occupy the even places?
9. A bag contains 4 red and 3 black balls. A second bag contains 2 red and 4 black balls. One bag is
selected at random. From the selected bag, one ball is drawn. Find the probability that the ball drawn is
red?
10. How many different four letter words can be formed (the words need not be meaningful) using the
letters of the word MEDITERRANEAN such that the first letter is E and the last letter is R?

Mensuration:
Mensuration Important Formulae:
Circle:
1. Diameter, D = 2r (r - Radius)
2
2. Area = πr sq. units
3. Circumference = 2πr units

Square:
2
4. Area = a sq. units (a – side of the square)
5. Perimeter = 4a units
6. Diagonal, d = √2 a units
Cuboid:
7. Volume = (Cross section area ´ height) = lbh cubic units
8. Lateral Surface Area (Area of the four walls) = 2h(l+b) sq. units
9. Total surface area (TSA) = 2(lb+bh+hl) sq. units
Cube:
3
10. Volume = a cubic units (a – side of the cube)
2
11. Lateral Surface Area = 4 a sq. units
2
12. Total Surface Area = 6a sq. units
13. Length of diagonal = a√3

Sphere:
3
14. Volume = (4/3) πr cubic units (r – radius of the sphere)
2
15. Surface Area = 4πr sq. units
Hemisphere:
3
16. Volume = (2/3)πr cubic units (r – radius of the hemisphere)
2
17. Total Surface Area = 3πr sq. units
2
18. Curved Surface Area = 2πr sq. units

Cylinder:
2
19. Volume = πr h cubic units (r – radius of the cylinder)
20. Curved surface Area or CSA = 2πrh sq. units(h – height of the cylinder)
2
21.TSA = CSA+ Areas of the top and bottom = 2πrh + 2πr =2πr(r+h) sq. units

Cone:
2
22. Volume = (1/3) πr h cubic Units) (r – radius of the cone)
r 2  h2
23. Slant height l=
24. Curved surface area= πrl sq. units
25. Total surface area= πr(r+l) sq. units.
Rectangle:
26. Perimeter= 2(l+b) units
27. Area = l x b sq. units
1. Find the curved surface area of a hemisphere whose diameter is 14 cm.
Explanation: Radius of hemisphere = 1/2 x 14 cm = 7 cm.
Therefore, Curved surface Area = 2πr2 = 2 x (22/7) x 7 x 7 = 308 cm 2.
2. Find the length of the longest pole that can be placed in a room 12 metre long, 8 metre broad and 9
metre high.
Explanation: Diagonal of room = Square root (l2 + b2 + h2)
= Square root ( 122 + 82 + 92)
= 17 metre.
Therefore, Length of longest pole = 17 metre
3. How much metal is required to make a 20 metre long pipe, if its inner and outer diameter are 6 metre
and 8 metre respectively.
Explanation: Outer and inner radius of the pipe are 4 metre and 3 metre respectively.
Therefore, Quantity of metal required
= Outer volume of the pipe - Inner volume of the pipe
= π(R2−r2)h.
= (22/7)×(16−9) x 20
= 440 m3.
4. A wire is drawn from a solid iron sphere of radius 6cm. Find the length of the wire, if its diameter is 6
cm.
Explanation: An iron wire is a form of cylinder.
Therefore, Volume of cylinder = Volume of Sphere
Therefore, π(Radius of cylinder)2 (Height of cylinder) = (4/3)π (Radius of sphere)3
Therefore, Height of the cylinder = [(4/3)x π(Radius of sphere)3 ]/[π(Radius of cylinder)2]
= (4/3×6×6×6)/(3×3)
= 32 cm
Therefore, Length of the wire is 32 cm.
5. A field is 90 metre long and 50 metre broad. A 25 metre long, 20 metre broad and 4 metre deep tank
dug in the field and the earth taken out is spread evenly over the remaining field. How much the level of
field will rise?
Explanation: Area of field = 90 x 50 = 4500 m2
Area of field dug out = 25 x 20 = 500 m2
Therefore, Area of remaining field = 4500 m 2 - 500 m2 = 4000 m2
Volume of the earth dug out = 25 x 20 x 4 = 2000 m 3
Therefore, Field will rise by 2000/4000 = 0.5 metre.
6. The area of the base of a right circular cone is 154 cm2 and its height is 14 cm. The curved surface of
the cone is?
Explanation: (22/7)×r2=154⇒r2=(154×722) = 49 ; r = 7 cm
Now, r = 7 and h = 14.
So,Slant height (l) = square root [(7)2+(14)2]
= 7√5
Area of curved surface = πrl = (22/7×7×7√5)cm2=154√5 cm2
7. A field is 40 metre long and 35 metre wide. The field is surrounded by a path of uniform width of 2.5
Explanation: metre runs round it on the outside. Find the area of the path.
Remember the formula for the Area of path = 2 x Width x [Length + Breadth + (2 x Width)]
= 2 x 25 x (40 + 35 + 2 x 2.5)
= 5 x (75 + 5) = 400 m2.
8. Find area of uniform path of width 2 metre running from centre of each side of the opposite side of a
rectangle field measuring 17 metre by 12 metre.
Explanation: Remember the formula for the Area of path
= Width of path x (Length of field + Breadth of field) - (Width of path)2
= 2 x (17 + 12) - (2)2
= 58 - 4 = 54 m2.
9. The diameter of a circle is 105 cm less than the circumference. What is the diameter of the circle?
Explanation: (πd−d)=105⇒(π−1)d=105
⇒(22/7−1)d = 105
d = (7/15×105)cm=49 cm.
10. The length of a rectangle is twice its breadth. If its length is decreased by 5 cm and the breadth is
increased by 5 cm, the area of the rectangle is increased by 75 cm2. Therefore, the length of the
rectangle is ?
Explanation: Let breadth = x cm and length = (2x) cm
Then. (2x-5)(x+5) - x×2x=75.
2x2+5x−25−2x2=75 or 5x = 100
or x = 20
Length = (2x)cm = 40 cm.
11. A person walked diagonally across a square plot. Approximately, what was the percent saved by not
walking along the edges?
Explanation:

Consider a square plot as shown above and let the length of each side = 1
Then length of the diagonal = √12+12=√2

Distance travelled if walked along the edges = BC + CD = 1 + 1 = 2


Distance travelled if walked diagonally = BD = √2=1.41
Distance Saved = 2 - 1.41 = 0.59

Percent distance saved = (0.59/2)×100=0.59×50≈30%.


12. The length of a rectangular plot is 20 metres more than its breadth. If the cost of fencing the plot
@Rs. 26.50 per metre is Rs. 5300, what is the length of the plot in metres?
Explanation: Length of the plot is 20 metres more than its breadth.
Hence, let's take the length as l metres and breadth as (l - 20) metres

Length of the fence = perimeter = 2(length + breadth)= 2[ l + (l - 20) ] = 2(2l - 20) metres
Cost per meter = Rs. 26.50
Total cost = 2(2l - 20) × 26.50

Total cost is given as Rs. 5300


=> 2(2l - 20) × 26.50 = 5300
=> (2l - 20) × 26.50 = 2650
=> (l - 10) × 26.50 = 1325
=> (l - 10) = (1325/26.50) = 50
=> l = 50 + 10 = 60 metres.
Exercise:
1. An error 2% in excess is made while measuring the side of a square. What is the percentage of error in
the calculated area of the square?
2. A rectangular park 60 m long and 40 m wide has two concrete crossroads running in the middle of the
park and rest of the park has been used as a lawn. The area of the lawn is 2109 sq. m. what is the width
of the road?
3. A towel, when bleached, lost 20% of its length and 10% of its breadth. What is the percentage of
decrease in area?
4. A rectangular parking space is marked out by painting three of its sides. If the length of the unpainted
side is 9 feet, and the sum of the lengths of the painted sides is 37 feet, find out the area of the parking
space in square feet?
5. The area of a rectangle plot is 460 square metres. If the length is 15% more than the breadth, what is
the breadth of the plot?
6. The length of a room is 5.5 m and width is 3.75 m. What is the cost of paying the floor by slabs at the
rate of Rs. 800 per sq. metre?
7. The ratio between the length and the breadth of a rectangular park is 3 : 2. If a man cycling along the
boundary of the park at the speed of 12 km/hr completes one round in 8 minutes, then what is the area
of the park (in sq. m)?
8. A tank is 25 m long, 12 m wide and 6 m deep. What is the cost of plastering of its walls and bottom at
the rate of 75 paise per sq. m?
9. If the diagonals of a rhombus are 24 cm and 10 cm, what will be its perimeter?
10. The diagonal of a rectangle is √41 cm and its area is 20 sq. cm. What is the perimeter of the
rectangle?
Calendars:
1. Odd Days:
We are supposed to find the day of the week on a given date.
For this, we use the concept of 'odd days'.
In a given period, the number of days more than the complete weeks are called odd days.
2. Leap Year:
(i). every year divisible by 4 is a leap year, if it is not a century.
(ii). every 4th century is a leap year and no other century is a leap year.
Note: A leap year has 366 days.
Examples:
i. Each of the years 1948, 2004, 1676 etc. is a leap year.
ii. Each of the years 400, 800, 1200, 1600, 2000 etc. is a leap year.
iii. None of the years 2001, 2002, 2003, 2005, 1800, 2100 is a leap year.
3. Ordinary Year:
The year which is not a leap year is called an ordinary years. An ordinary year has 365 days.
4. Counting of Odd Days:
One ordinary year = 365 days = (52 weeks + 1 day.)
One ordinary year has 1 odd day.
One leap year = 366 days = (52 weeks + 2 days)
One leap year has 2 odd days.
i. 100 years = 76 ordinary years + 24 leap years
= (76 x 1 + 24 x 2) odd days = 124 odd days.
= (17 weeks + days) 5 odd days.
Number of odd days in 100 years = 5
Number of odd days in 200 years = (5 x 2) 3 odd days.
Number of odd days in 300 years = (5 x 3) 1 odd day.
Number of odd days in 400 years = (5 x 4 + 1) 0 odd day.
Similarly, each one of 800 years, 1200 years, 1600 years, 2000 years etc. has 0 odd days.
2. Day of the Week Related to Odd Days:
No. of
0 1 2 3 4 5 6
days:
Su Mo Tues We Thur Fr Sat
Day:
n. n. . d. s. i. .

Examples:
1. What was the day of the week on 20 may, 1985?

Solution: Here Number of odd days in 1600 years = 0

Number of odd days in 300 years from 1600 to 1900 = 5*3 = 2 week + 1 odd day= 1 odd day

Number of odd days in 84 years= 21 leap year + 63 days = 21*2 + 63*1 = 105 days = 0 odd days

Number of odd days in 20 may = 31 days of Jan. + 28 days of Feb. + 31 days of mar. + 30 days in April + 20
days in May = 140 days = 0 odd day

so, total number of odd days = 0+1+0+0=1 = Monday.


2. Prove that calendar for the year 2009 will serve for the year 2015.

Solution: for this sum of odd days from 2009 to 2014 should be zero.
Year 2009 2010 2011 2012 2013 2014
Odd day 1 1 1 2 1 1

Sum of odd days = 1+1+1+2+1+1=7= 1 week + 0 odd days


So, both dates 1.1.2009 and 1.1.2015 will be on same day , so calendar for the year 2009 will serve for
the year 2015.
3. On what date of Feb. 2007 did Saturday fall?

Solution: For this find the day of 1.2.2007

1600+400 years has 0 odd days

From 2001 to 2006 there are 1 leap years + 5 ordinary years

So number of odd days = 1*2 + 5*1 = 2 + 5 = 7 = 1 week = 0 odd day

Now from 1.1.2007 to 1.2.2007 number of days = 32 = 4 weeks + 4 odd days = 4 odd days

So, total number of odd days = 4, so 1.2.2007 will be Thursday

Now Saturday will be on 3.2.2007


4. Today is Tuesday. After 72 days, it will be?

Solution: Tuesday will be repeated after each 7 days so, after at 70 day it will also be Tuesday, so at 72 th
day it will be Friday.
5. The last day of a century cannot be?
Solution:

6. It was Sunday on Jan 1, 2006. What was the day of the week Jan 1, 2010?
Solution:
On 31st December, 2005 it was Saturday.
Number of odd days from the year 2006 to the year 2009 = (1 + 1 + 2 + 1) = 5 days.
On 31st December 2009, it was Thursday.
Thus, on 1st Jan, 2010 it is Friday.
7. If 6th March, 2005 is Monday, what was the day of the week on 6th March, 2004?
Solution: The year 2004 is a leap year. So, it has 2 odd days.
But, Feb 2004 not included because we are calculating from March 2004 to March 2005.
So it has 1 odd day only.
The day on 6th March, 2005 will be 1 day beyond the day on 6th March, 2004.
Given that, 6th March, 2005 is Monday.
6th March, 2004 is Sunday (1 day before to 6th March, 2005).
8. What was the day of the week on 28th May, 2006?
Solution: 28 May, 2006 = (2005 years + Period from 1.1.2006 to 28.5.2006)
Odd days in 1600 years = 0
Odd days in 400 years = 0
5 years = (4 ordinary years + 1 leap year) = (4 x 1 + 1 x 2) 6 odd days
(31[Jan] + 28 [Feb]+ 31[Mar] + 30[April] + 28[May] ) = 148 days
148 days = (21 weeks + 1 day) 1 odd day.
Total number of odd days = (0 + 0 + 6 + 1) = 7 0 odd day.
Given day is Sunday

Exercise:
1. on what dates of April, 2001 did Wednesday fall?
2. What was the day of the week on, 16th July, 1776?
3. What was the day of the week on 17th June, 1998?
4. On 8th Dec, 2007 Saturday falls. What day of the week was it on 8th Dec, 2006?
5. If Aug 15th, 2012 falls on Thursday then June 11th, 2013 falls on which day?
6. The calendar for the year 2007 will be the same for the year?
7. The calendar of the year 2024 can be used again in the year?
8. What will be the day of the week 15 th August, 2010?

You might also like